Está en la página 1de 60

MISCELLANEOUS

1. To determine the greatest possible number of contributors we must assume that each of these individuals contributed the minimum amount, or $50. We can then set up an inequality in which n equals the number of contributors 50n is less than or equal to $1,!"# $ivide both sides of the equation by 50 to isolate n, and get n is less than or equal to %".#& 'ince n represents individual people, it must be the greatest whole number less than %".#&. Thus, the greatest possible value of n is %". (lternately, we could have assumed that the fundraiser collected $1,!50 rather than $1,!"#. )f it had, and we assumed each individual contributed the minimum amount, there would have been e*actly %5 contributors +$50 * %5 , $1,!50-. 'ince the fundraiser actually raised one dollar less than $1,!50, there must have been one fewer contributor, or %". The correct answer is .. /. )t may be easiest to represent the ages of 0oan, 1ylie, 2illian and 3iriam +J, K, L and Mon a number line. )f we do so, we will see that the ages represent consecutive integers as shown in the diagram.

'ince the ages are consecutive integers, they can all be e*pressed in terms of L L, L 4 1, L 4 /, L 4 %. The sum of the four ages then would be "L 4 5. 'ince L must be an integer +it6s 2illian6s age-, the e*pression "L 4 5 describes a number that is two more than a multiple of " "L 4 5 , +"L 4 "- 4 / 7"L 4 " describes a multiple of ", since it can be factored into "+L 4 1- or " 8 an integer.9 5" is the only number in the answer choices that is two more than a multiple of " +namely, 5/-. The correct answer is $. %. This is an algebraic translation problem dealing with ages. :or this type of problem, an age chart can help us ;eep trac; of the variables <=W )< 5 >?(@'

0(<?T A(@=2

J C

J45 C45

Bsing the chart in combination with the statements given in the question, we can derive equations to relate the variables. The first statement tells us that 0anet is now /5 years younger than her mother Aarol. 'ince we have used J to represent 0anet6s current age, and C to represent Aarol6s current age, we can translate the statement as follows J , C C /5. The second statement tells us that 0anet will be half Aarol6s age in 5 years. 'ince we have used (J 4 5- to represent 0anet6s age in 5 years, and +C 4 5- to represent Aarol6s age in 5 years, we can translate the statement as follows J 4 5 , +1D/-+C 4 5-. <ow, we can substitute the e*pression for C +C , J 4 /5- derived from the first equation into the second equation +note we choose to substitute for C and solve for J because the question as;s us for 0anetEs age 5 years agoJ 4 5 , +1D/-+J 4 /5 4 5J + 6 , +1D/-+J 4 %1/J 4 1/ , J 4 %1 J , 1# )f 0anet is now 1# years old, she was 1" years old 5 years ago. The correct answer is .. ". The $1,""0 is divided into 1/ equal monthly allocations. 1""0D1/ , $1/0 The company has $1/0 allocated per month for entertainment, so the allocation for three months is 1/0 F % , %50 'ince the company has spend a total of $%00 thus far, it is $%50 G $%00 , $50 under budget. The correct answer is (. 5. 'ince this problem includes variables in both the question and the answer choices, we can try solving by plugging in smart numbers. :or *, we want to choose a multiple of / because we will have to ta;e *D/ later. 2etEs say that (A3? produces " brooms per month from 0anuary to (pril, so * , ". The total number of brooms produced was +" brooms * " months-, or 15 brooms. (A3? sold *D/ brooms per month, or / brooms per month +because we chose * , "-. <ow we need to start figuring out the storage costs from 3ay /nd to $ecember %1st. 'ince (A3? sold / brooms on 3ay 1st, it needed to store 1" brooms that month, at a cost of

$1". :ollowing the same logic, we see that (A3? sold another two brooms 0une 1st and stored 1/ brooms, which cost the company $1/. We now see that the 0uly storage costs were $10, (ugust were $&, 'eptember $5, =ctober $", <ovember $/, and for $ecember there were no storage costs since the last / brooms were sold on $ecember 1st. 'o (A3?Es total storage costs were 1" 4 1/ 4 10 4 & 4 5 4 " 4 / , $55. <ow we Hust need to find the answer choice that gives us $55 when we plug in the same value, * , ", that we used in the question. 'ince 1" * " , 55, $1"* must be the correct value. The correct answer is ?. While plugging in smart numbers is the preferred method for I)A problems such as this one, it is not the only method. .elow is an alternative, algebraic method for solving this problem (A3? accumulated an inventory of "x brooms during its fourGmonth production period. )f it sold 0.5x brooms on 3ay 1st , then it paid storage for %.5x brooms in 3ay, or $%.5x. (gain, if (A3? sold 0.5x brooms on 0une 1st, it paid storage for %x brooms in 0une, or $%x. The first row of the table below shows the amount of money spent per month on storage. <otice that since (A3? liquidated its stoc; on $ecember 1st, it paid Jero dollars for storage in $ecember. 3(> $%.5x 0B< $%x 0B2 $/.5x (BK $/x '?L $1.5x =AT $1x <=I $0.5x

)f we add up these costs, we see that (A3? paid $1"x for storage. 5. The bus will carry its greatest passenger load when P is at its ma*imum value. )f P , G/+S C "-/ 4 %/, the ma*imum value of P is %/ because +S C "-/ will never be negative, so the e*pression G/+S C "-/ will never be positive. The ma*imum value for P will occur when G/+S C "-/ , 0, i.e. when S , ". The question as;s for the number of passengers two stops after the bus reaches its greatest passenger load, i.e. after 5 stops +S , 5-. P , G/+5 C "-/ 4 %/ P , G/+/-/ 4 %/ P , G& 4 %/ P , /" The correct answer is A. (lternatively, the ma*imum value for P can be found by building a table, as follows S 0 1 / % P 0 1" /" %0

" 5 5

%/ %0 /"

The ma*imum value for P occurs when S , ". Thus, two stops later at S , 5, P , /". (nswer choice A is correct. !. 0ohn was /! when he married .etty, and since they Hust celebrated their fifth wedding anniversary, he is now %/. 'ince .ettyEs age now is !D& of 0ohnEs, her current age is +!D&- F %/, which equals /&. The correct answer is A. &. 0oe uses 1D" of %50, or #0 gallons, during the first wee;. Me has /!0 gallons remaining +%50 C#0 , /!0-. $uring the second wee;, 0oe uses 1D5 of the remaining /!0 gallons, which is 5" gallons. Therefore, 0oe has used 1"" gallons of paint by the end of the second wee; +#0 4 5" , 1""-. The correct answer is .. #. =ne way to do this problem is to recogniJe that the star earned $&3 more +$%/3 G $/"3 , $&3- when her film grossed $"03 more +$1003 G $503 , $"03-. 'he wants to earn $"03 on her ne*t film, or $&3 more than she earned on the more lucrative of her other two films. Thus, her ne*t film would need to gross $"03 more than $1003, or $1"03. (lternatively, we can solve this problem using algebra. The starEs salary consists of a fi*ed amount and a variable amount, which is dependent on the gross revenue of the film. We ;now what she earned for two films, so we can set up two equations, where f is her fi*ed salary and p is her portion of the gross, e*pressed as a decimal 'he earned $%/ million on a film that grossed $100 million $%/3 , f 4 p+$1003'he earned $/" million on a film that grossed $50 million $/"3 , f 4 p+$503We can solve for p by subtracting the second equation from the first $%/3 , f 4 p+$1003C 7$/"3 , f 4 p+$503-9 $&3 , p+$"030./ , p We can now plug in 0./ for p in either of the original equations to solve for f $%/3 , f 4 0./+$1003$%/3 , f 4 $/03 $1/3 , f <ow that we ;now her fi*ed salary and the percentage of the gross earnings she

receives, we can rewrite the formula for her total earnings as Total earnings , $1/3 4 0./+gross:inally, we Hust need to figure out how much gross revenue her ne*t film needs to generate in order for her earnings to be $"0 million $"03 , $1/3 4 0./+gross$/&3 , 0./+gross$/&3D0./ , $1"03 , gross The correct answer is $. 10. ). B<A?@T()< )t depends on how many bicycles <orman sold. :or e*ample, if x , ", then <orman earned $"" 7, $/0 4 +" F $5-9 last wee;. )n order to double his earnings, he would have to sell a minimum of # bicycles this wee; +y , #-, ma;ing $#/ 7, $/0 4 +5 F $5- 4 +% F $1/-9. )n that case, y N /x. Mowever, if x , 5 and y , 11, then <orman would have earned $55 7, $/0 4 +5 F $5-9 last wee; and $115 7, $/0 4 +5 F $5- 4 +5 F $1/-9 this wee;. )n that case, $115 N / F $55, yet y O /x . 'o, it is possible for <orman to more than double his earnings without selling twice as many bicycles. )). T@B? )n order to earn more money this wee;, <orman must sell more bicycles. ))). T@B? )f <orman did not sell any bicycles at all last wee; +x , 0-, then he would have earned the minimum fi*ed salary of $/0. 'o he must have earned at least $"0 this wee;. )f y , %, then <orman earned $%& 7, $/0 4 +% F $5-9 this wee;. )f y , ", then <orman earned $"" 7, $/0 4 +" F $5-9 this wee;. Therefore, <orman must have sold at least " bicycles this wee;, which can be e*pressed y N %. The correct answer is $. 11. )n order to determine the greatest number of points that an individual player might have scored, assume that 11 of the 1/ players scored ! points, the minimum possible. The 11 low scorers would therefore account for !+11- , !! points out of 100. The number of points scored by the twelfth player in this scenario would be 100 C !! , /%. The correct answer is ?. 1/. 'ince we are not given any actual spending limits, we can pic; numbers. )n problems involving fractions, it is best to pic; numbers that are multiples of the denominators. We can set the spending limit for the gold account at $15, and for the platinum card at $%0. )n this case, 'ally is carrying a balance of $5 +which is 1D% of $15- on her gold card, and a balance of $5 +1D5 of $%0- on her platinum card. )f she transfers the balance from her gold card to her platinum card, the platinum card will have a balance of $11. That means that $1# out of her $%0 spending limit would remain unspent.

(lternatively, we can solve this algebraically by using the variable x to represent the spending limit on her platinum card +1D5-x 4 +1D%-+1D/-x , +1D5-x 4 +1D5-x , +5D%0-x 4 +5D%0-x , +11D%0-x This leaves 1#D%0 of her spending limit untouched. The correct answer is $. 13. The problem tal;s about 3artina and LamEs incomes but never provides an actual dollar value, either in the question or in the answer choices. We can, therefore, use smart numbers to solve the problem. .ecause the dollar value is unspecified, we pic; a dollar value with which to solve the problem. To answer the question, we need to calculate dollar values for the portion of income each earns during the ten months not including 0une and (ugust, and we also need to calculate dollar values for each playerEs annual income. 2etEs start with 3artina, who earns 1D5 of her income in 0une and 1D& in (ugust. The common denominator of the two fractions is /", so we set 3artinaEs annual income at $/". This means that she earns $" +1D5 F /"- in 0une and $% +1D& F /"- in (ugust, for a total of $! for the two months. )f 3artina earns $! of $/" in 0une and (ugust, then she earns $1! during the other ten months of the year. The problem tells us that Lam earns the same dollar amount during the two months as 3artina does, so Lam also earns $! for 0une and (ugust. The $! Lam earns in 0une and (ugust represents 1D% 4 1D" of her annual income. To calculate her annual income, we solve the equation ! , +1D% 4 1D"-x, with x representing LamEs annual income. This simplifies to ! , +!D1/-x or 1/ , x. )f Lam earns $! of $1/ in 0une and (ugust, then she earns $5 during the other ten months of the year. 7<=T? we cannot simply pic; a number for Lam in the same way we did for 3artina because we are given a relationship between 3artinaEs income and LamEs income. )t is a coincidence that LamEs income of $1/ matches the common denominator of the two fractions assigned to Lam, 1D% and 1D" G if we had pic;ed $"& for 3artinaEs income, LamEs income would then have to be $/", not $1/.9 Aombined, the two players earn $1! 4 $5 , $// during the other ten months, out of a combined annual income of $/" 4 $1/ , $%5. The portion of the combined income earned during the other ten months, therefore, is //D%5 which simplifies to 11D1&. <ote first that you can also calculate the portion of income earned during 0une and (ugust and then subtract this fraction from 1. The portion of income earned during 0une and (ugust, !D1&, appears as an answer choice, so be careful if you decide to solve it this way. <ote also that simply adding the four fractions given in the problem produces the number

!D&, an answer choice. 1D& +or 1 C !D&- is also an answer choice. These two answers are Ptoo good to be trueP G that is, it is too easy to arrive at these numbers. The correct answer is $. 1". This fraction problem contains an PunspecifiedP total +the x liters of water in the la;e-. Lic; an easy PsmartP number to ma;e this problem easier. Bsually, the smart number is the lowest common denominator of all the fractions in the problem. Mowever, if you pic; /&, you will quic;ly see that this yields some unwieldy computation. The easiest number to wor; with in this problem is the number ". 2etEs say there are " liters of water originally in the la;e. The question then becomes $uring which year is the la;e reduced to less than 1 liter of waterQ (t the end of /0!5, there are " F +5D!- or /0D! liters of water in the la;e. This is not less than 1. (t the end of /0!!, there are +/0D!- F +5D!- or 100D"# liters of water in the la;e. This is not less than 1. (t the end of /0!&, there are +100D"#- F +5D!- or 500D%"% liters of water in the la;e. This is not less than 1. (t the end of /0!#, there are +500D%"%- F +5D!- or /500D/"01 liters of water in the la;e. This is not less than 1. (t the end of /0&0, there are +/500D/"01- F +5D!- or 1/500D15&0! liters of water in the la;e. This is less than 1. <otice that pic;ing the number " is essential to minimiJing the computation involved, since it is very easy to see when a fraction falls below 1 +when the numerator becomes less than the denominator.- The only moderately difficult computation involved is multiplying the denominator by ! for each new year. The correct answer is $. 15. This fraction problem contains an unspecified total +the number of married couplesand is most easily solved by a pic;ing a PsmartP number for that total. The smart number is the least common denominator of all the fractions in the problem. )n this case, the smart number is /0. 2etEs say there are /0 married couples. 15 couples +%D" of the total- have more than one child. & couples +/D5 of the total- have more than three children. This means that 15 C & , ! couples have either / or % children. Thus !D/0 of the married couples have either / or % children. The correct answer is A. 15. We can bac; solve this question by using the answer choices. 2et6s first chec; to ma;e sure that each of the 5 possible prices for one candy can be paid using e*actly " coins & , 5414141 1% , 10414141 "0 , 10410410410 5% , 50414141 55 , 504104541

'o far we can6t ma;e any eliminations. <ow let6s chec; two pieces of candy 15 , 5 4 5 4 5 4 1 /5 , 10 4 10 4 5 4 1 &0 , /5 4 /5 4 /5 4 5 105 , 50 4 50 4 5 4 1 1%/ , 50 4 50 4 /5 4 5 4 1 4 1 We can eliminate answer choice ? here. <ow three pieces of candy /" , 10 4 10 4 1 4 1 4 1 4 1 %# , /5 4 10 4 1 4 1 4 1 4 1 1/0 , 50 4 50 4 10 4 10 15# , 50 4 50 4 50 4 5 4 1 4 1 4 1 4 1. We can eliminate answer choices (, . and $. <otice that at a price of "0R, .illy can buy four and five candies with e*actly " coins as well 150 , 50 4 50 4 50 4 10 /00 , 50 4 50 4 50 4 50 This problem could also have been solved using divisibility and remainders. <otice that all of the coins are multiples of 5 e*cept pennies. )n order to be able to pay for a certain number of candies with e*actly four coins, the total price of the candies cannot be a value that can be e*pressed as 5x 4 ", where x is a positive integer. )n other words, the total price cannot be a number that has a remainder of " when divided by 5. WhyQ The remainder of " would alone require " pennies. We can loo; at the answer choices now Hust focusing on the remainder when each price and its multiples are divided by 5 @emainder when price for 1 candy is divided by 5 % % 0 % 1 @emainder when price for / candies is divided by 5 1 1 0 1 / @emainder when price for % candies is divided by 5 " " 0 " % @emainder when price for " candies is divided by 5 1 / 0 / "

Lrice per candy & 1% "0 5% 55

The only price for which none of its multiples have a remainder of " when divided by 5 is "0R.

<otice that not having a remainder of " does not guarantee that e*actly four coins can be usedS however, having a remainder of " does guarantee that e*actly for coins cannot be usedT The correct answer is A.

17. :rom the question we ;now that "0 percent of the violetDgreen mi* is blue pigment. We also ;now that %0 percent of the violet paint and 50 percent of the green paint is blue pigment. 'ince the blue pigment in the violetDgreen mi* is the same blue pigment in the original violet and green paints, we can construct the following equation .%v 4 .5g , ."+v + g.%v 4 .5g , ."v 4 ."g .1g , .1v g v Therefore, the amount of violet paint is equal to the amount of green paint in the brown mi*ture, each contributing 50 percent of the total. 'ince the red pigment represents !0 percent of the weight of the violet paint, it must account for !0 percent of 50 percent of the weight of the brown mi*. This represents +.!-+.5- , .%5, or %5U of the total weight of the brown mi*. 'ince we have 10 grams of the brown paint, the red pigment must account for +.%5-+10- , %.5 grams of the brown paint. There is an alternative way to come up with the conclusion that there must be equal amounts of green and violet paints in the mi*. 'ince there is blue paint in both the violet and green paints, when we combine the two paints, the percentage of blue paint in the mi* will be a !eighted average of the percentages of blue in the violet paint and the percentage of blue in the green paint. :or e*ample, if there is twice as much violet as green in the brown mi*, the percentage of blue in the violet will get double weighted. :rom loo;ing at the numbers, however, "0U is e*actly the simple average of the %0U blue in violet and the 50U blue in green. This means that there must be an equal amount of both paints in the mi*. 'ince there are equal amounts of violet and green paint in the 10 grams of brown mi*ture, there must be 5 grams of each. The violet paint is !0U red, so there must be +.!-+5- , %.5 grams of red paint in the mi*. The correct answer is .. 18. This question requires us to untangle a series of ratios among the numbers of wor;ers in the various years in order to find the number of wor;ers after the first year. We can solve this problem by setting up a grid to ;eep trac; of the information .efore (fter >ear 1 (fter >ear / (fter >ear % (fter >ear "

We are told initially that after the fourGyear period, the company has 10,500 employees .efore (fter >ear 1 (fter >ear / (fter >ear % (fter >ear " 10,500 We are then told that the ratio of the number of wor;ers after the fourth year to the number of wor;ers after the second year is 5 to 1. This implies that the number of wor;ers after the fourth year is si* times greater than that after the second year. Thus the number of wor;ers after the second year must be 10,500D5 , 1,!50 .efore (fter >ear 1 (fter >ear / 1,!50 (fter >ear % (fter >ear " 10,500

We are then told that the ratio of the number of wor;ers after the third year to the number after the first year is 1" to 1. We can incorporate this into the chart .efore (fter >ear 1 x (fter >ear / 1,!50 (fter >ear % 1"x (fter >ear " 10,500

<ow we are told that the ratio of the number of wor;ers after the third year to that before the period began is !0 to 1. We can incorporate this into the chart as well .efore " (fter >ear 1 x (fter >ear / 1,!50 (fter >ear % 1"x !0y (fter >ear " 10,500

:rom the chart we can see that 1"x , !0y# Thus x , 5y .efore " (fter >ear 1 $y (fter >ear / 1,!50 (fter >ear % !0y (fter >ear " 10,500

'ince the ratio between consecutive years is always an integer and since after three years the number of wor;ers is !0 times greater, we ;now that the series of ratios for the first three years must include a /, a 5, and a ! +because / * 5 * ! , !0-. .ut this fact by itself does not tell us the order of the ratios. )n other words, is it / G 5 G ! or ! G / G 5 or 5 G / G !, etcQ We do ;now, however, that the factor of 5 is accounted for in the first year. 'o we need to ;now whether the number of wor;ers in the second year is twice as many or seven times as many as in the first year. @ecall that the number of wor;ers after the fourth year is si* times greater than that after the second year. This implies that the ratios for the third and fourth years must be / and % or % and /. This in turn implies that the ratio of ! to 1 must be between the first and second years. 'o 1,!50 is ! times greater than the number of wor;ers after the first year. Thus, 1,!50D! , /50. (lternatively, since the question states that the ratio between any two years is always an integer, we ;now that 1,!50 must be a multiple of the number of wor;ers after the first year. 'ince only !0 and /50 are factors of 1!50, we ;now the answer must be either

choice . or choice A. )f we assume that the number of wor;ers after the first year is !0, however, we can see that this must cannot wor;. The number of wor;ers always increases from year to year, but if !0 is the number of wor;ers after the first year and if the number of wor;ers after the third year is 1" times greater than that, the number of wor;ers after the third year would be 1" * !0 , #&0, which is less than the number of wor;ers after the second year. 'o choice . is eliminated and the answer must be choice A. The correct answer is choice A /50. 19. )t is important to remember that if the ratio of one group to another is x y, the total number of obHects in the two groups together must be a multiple of x 4 y. 'o since the ratio of rams to ewes on the farm is " to 5, the total number of sheep must be a multiple of # +" parts plus 5 parts-. (nd since the ratio of rams to ewes in the first pen is " to 11, the total number of sheep in the first pen must be a multiple of 15 +" parts plus 11 parts-. 'ince the number of sheep in each pen is the same, the total number of sheep must be a multiple of both # and 15. )f we assume that the total number of sheep is "5 +the lowest common multiple of # and 15-, the number of rams is /0 and the number of ewes is /5 +ratio " 5-. "5D% , 15, so there are 15 sheep in each pen. Therefore, there are " rams and 11 ewes in the first pen +ratio " 11-. This leaves /0 G " , 15 rams and /5 G 11 , 1" ewes in the other two pens. 'ince the second and third pens have the same ratio of rams to ewes, they must have 15D/ , & rams and 1"D/ , ! ewes each, for a ratio of & ! or &D!. (lternatively, we can answer the question algebraically. 'ince the ratio of rams to ewes in the first pen is " 11, let the number of rams in the first pen be "x and the number of ewes be 11x. 2et r be the number of rams in the second pen and let e be the number of ewes in the second pen. 'ince the number of sheep in each pen is the same, we can construct the following equation "x 4 11x , r 4 e, or 15x , r 4 e. 'ince the number of sheep in each pen is the same, we ;now that the number of rams in the second and third pens together is /r and the number of ewes in the second and third pens together is /e. Therefore, the total number of rams is "x 4 /r. The total number of ewes is 11x 4 /e. 'ince the overall ratio of rams to ewes on the farm is " 5, we can construct and simplify the following equation

We can find the ratio of r to e by setting the equations we have equal to each other. :irst, though, we must multiply each one by coefficients to ma;e them equal the same value

'ince both equations now equal 1/0x, we can set them equal to each other and simplify

The correct answer is (. 20. We can find a ratio between the rates of increase and decrease for the corn and wheat

To get rid of the radical sign in the denominator, we can multiply top and bottom by and simplify

This ratio indicates that for every cent that the price of wheat decreases, the price of corn increases by corn will increase by cents. 'o if the price of wheat decreases by x cents, the price of cents.

'ince the difference in price between a pec; of wheat and a bushel of corn is currently $/.50 or /50 cents, the amount by which the price of corn increases plus the amount by which the price of wheat decreases must equal /50 cents. We can e*press this as an equation %&ount Corn 'n(reases 4 %&ount )heat *e(reases , /50

We can then rewrite this word equation using variables. 2et ( be the decrease in the price of wheat in cents

<otice that the radical / was replaced with its appro*imate numerical value of 1." because the question as;s for the appro*imate price. We need not be e*act in this particular instance. )f ( , /0, we ;now that the price of a pec; of wheat had decreased by /0 cents when it reached the same level as the increased price of a bushel of corn. 'ince the original price of a pec; of wheat was $5.&0, its decreased price is $5.&0 G $0./0 , $5.50. +.y the same to;en, since ( , /0, the price of a bushel of corn had increased by /0+ - cents when it reached the same level as the decreased price of a pec; of wheat. This is equivalent to an increase of appro*imately /"0 cents. Thus the increased price of a bushel of corn , $%./0 4 $/."0 , $5.50.The correct answer is ?. /1. 2et s represent the number of science maHors, & represent the number of math maHors, h represent the number of history maHors, and l represent the number of linguistics maHors. We can set up the following equations s , +1D%+h & , +/D%-h s 4 & 4 h 4 l , /000 We can substitute and isolate the number of linguistics maHors. +1D%+h 4 +/D%+h 4 h 4 l , /000 /h 4 l , /000 l, /000 C /h We can rephrase the question PMow many students maHor in historyQP

+1- 'B::)A)?<T )f l , &, and & , +/D%-h, we can solve for h +1D%-h 4 +/D%-h 4 h 4 l , /000 +1D%+h 4 +/D%+h 4 h 4+/D%-h , /000 +&D%-h , /000 h , /000+%D&h , !50 )f h , !50, l , +/D%+h , 500. +/- 'B::)A)?<T )f & , s 4 /50, and & , +/D%-h and s , +1D%-h, we can substitute and solve for h +/D%-h , +1D%-h 4 /50 +1D%+h , /50 h , !50 )f h , !50, l , /000 C /+!50- , 500. The correct answer is $. 22. We can thin; of the liquids in the red buc;et as liquids (, ., A and ?, where ? represents the totality of every other ,ind of li-uid that is not %. /. or C# )n order to determine the percentage of ? contained in the red buc;et, we will need to determine the total amount of ( 4 . 4 A and the total amount of ?. 't is T0MPT'12 (3ut in(orre(t+ to use the follo!ing logi( !ith the infor&ation given in State&ent (1+# 'tatement +1- tells us that the total amount of liquids (, ., and A now in the red buc;et is 1./5 times the total amount of liquids ( and . initially contained in the green buc;et. 2etEs begin by assuming that, initially, there are 10 ml of liquid ( in the green buc;et. Bsing the percentages given in the problem we can now determine that the composition of the green buc;et was as follows 10U ( , 10 ml 10U . , 10 ml &0U ? , &0 ml 'ince there were /0 total ml of ( and . in the green buc;et, we ;now from statement +1that there must be /5 ml of ( 4 . 4 A now in the red buc;et +since /5 is 1./5 times /0-. :rom this we can deduce that, there must have been 5 ml of A in the blue buc;et. We can use the percentages given in the problem to determine the e*act initial composition of the blue buc;et 10U A , 5 ml #0U ? , "5 ml 'ince the liquid in the red buc;et is simply the totality of all the liquids in the green buc;et plus all the liquids in the blue buc;et, we can use this information to determine the total amount of ( 4 . 4 A +/5 ml- and the total amount of ? +&0 4 "5 , 1/5 ml- in the red

buc;et. Thus, the percentage of liquid now in the red buc;et that is <=T (, ., or A is equal to 1/5D150 , &% 1D% percent. This ratio +or percentage- will always remain the same no matter what initial amount we choose for liquid ( in the green buc;et. This is because the relative percentages are fi*ed. We can generaliJe that given an initial amount * for liquid ( in the green buc;et, we ;now that the amount of liquid . in the green buc;et must also be * and that the amount of ? in the green buc;et must be &*. We also ;now that the amount of liquid A in the blue buc;et must be .5*, which means that the amount of ? in the blue buc;et must be ".5*. Thus the total amount of ( 4 . 4 A in the red buc;et is * 4 * 4 .5* , /.5* and the total amount of liquid ? in the red buc;et is &* 4 ".5* , 1/.5*. Thus the percentage of liquid now in the red buc;et that is <=T (, ., or A is equal to 1/.5*D15* or &% 1D% percent. 4o!ever. the a3ove logi( is 5L%)0* 3e(ause it assu&es that the green 3u(,et does not (ontain li-uid C and that the 3lue 3u(,et does not (ontain li-uids % or /# )n other words, the above logic assumes that ;nowing that there are * ml of ( in the green buc;et implies that there are &* ml of ? in the green buc;et. @emember, however, that ? is defined as the totality of every li-uid that is 16T %. /. or C7 While the problem gives us information about the percentages of ( and . contained in the green buc;et, it does not tell us anything about the percentage of A contained in the green buc;et and we cannot Hust assume that this is 0. )f the percentage of A in the green buc;et is not 0, then this will change the percentage of ? in the green buc;et as well as changing the relative amount of liquid A in the blue buc;et. :or e*ample, letEs say that the green buc;et contains 10 ml of liquids ( and . but also contains % ml of liquid A. Ta;e a loo; at how this changes the logic Kreen buc;et 10U ( , 10 ml 10U . , 10 ml %U A , % ml !!U ? , !! ml 'ince there are /0 total ml of ( and . in the green buc;et, we ;now from statement +1that there must be /5 ml of ( 4 . 4 A in the red buc;et +since /5 is 1./5 times /0-. 'ince the green buc;et already contributes /% ml of this total, we ;now that there must be / total ml of liquids (, . and A in the blue buc;et. )f the blue buc;et does not contain liquids ( or . +which we cannot necessarily assume-, then the composition of the blue buc;et would be the following 10U A , / ml #0U ? , 1& ml <ote, however, that if the blue buc;et does contain some of liquids ( or ., then the composition of the blue buc;et might also be the following 10U A , 1 ml 10U ( , 1 ml &0U ? , & ml <otice that it is impossible to ascertain the e*act amount of ? in the red buc;et G since this amount will change depending on whether the green buc;et contains liquid A andDor the blue buc;et contains liquids ( or .. Thus statement +1- by itself is <=T sufficient to answer this question. 'tatement +/- tells us that the green and blue buc;ets did not contain any of the same liquids. (s such, we ;now that the green buc;et did not contain liquid A and that the blue buc;et did not contain liquids ( or .. =n its own, this does not help us to answer the question. Mowever, ta;ing 'tatement +/- together with 'tatement +1-, we can definitively

answer the question. The correct answer is A .=TM statements T=K?TM?@ are sufficient, but <?)TM?@ statement (2=<? is sufficient. 23. When solve such ;ind of questions, we Hust need to ;now the ratio one price to another price. )t is time waste to calculate one by one. .oth two statements do not give the information, as well as their combination. (nswer is ? /". This question can be restated in several ways. 2et )or, , amount earned +i.e., amount needed to purchase the Hac;et-. @ecall, )or, , 8ate * Ti&e. 'ince the number of hours that either 0im or Tom need to wor; in order to purchase the Hac;et is given, we need only ;now either personEs rate of pay to determine the cost of the Hac;etS hence, the question can be restated as either PWhat is xQP or PWhat is yQP. (lso, since the amount of time needed for either 0im or Tom to purchase the Hac;et is given, it can be shown that the amount of time needed for them wor;ing together to purchase the Hac;et can also be calculated. The formula )or, , 8ate * Ti&e also applies when 0im and Tom wor; togetherS hence, only the combined rate of 0im and Tom wor;ing together is required. 'ince the combined rate of two people wor;ing together is equal to the sum of their individual rates, the question can also be restated as PWhat is 9 + "QP +1- )<'B::)A)?<T This statement gives only the relative earning power of 0im and Tom. 'ince the original question states the amount of time needed for either 0im or Tom to earn enough money to purchase the Hac;et, it also gives us the relative earning power of 0im and Tom. Mence, statement +1- does not add any information to the original question. +/- 'B::)A)?<T 2et : , 1 Hac;et. 'ince Tom and 0im must " and 5 hours, respectively, to earn enough to buy 1 Hac;et, in units of PHac;et per hour,P 0im wor;s at the rate of 1D" Hac;ets per hour and Tom wor;s at the rate of 1D5 Hac;ets per hour. Their combined rate is 1D" 4 1D5 , 5D/0 4 "D/0 , #D/0 Hac;ets per hour. 'ince Ti&e , )or,;8ate, Ti&e , 1 Hac;etD+#D/0 Hac;ets per hour- , /0D# hours. 'ince the combined pay rate of the 0im and Tom is equal to the sum of the individual pay rates of the twoS hence, the combined pay rate in dollars per hour is 9 + ". When the two wor; together, %&ount0arned , Co&3inedPay8ate * Ti&e , +9 + "- * #D/0. 'ince statement +/- states that 9 + " , $"%.!5, this statement is sufficient to compute the cost of the Hac;et +it is not necessary to ma;e the final calculation-. The correct answer is .. <ote )t is also not necessary to e*plicitly compute the time needed for 0im and Tom wor;ing together to earn the Hac;et +/0D# hours-. )t is only necessary to recogniJe that this number (an 3e (al(ulated in order to determine that +/- is sufficient.

/5. The question stem tells us that .ill has a stac; of $1, $5, and $10 bills in the ratio of 10 5 1 respectively. WeEre trying to find the number of $10 bills. +1- )<'B::)A)?<T 'ince the ratio of the number of $1 bills to $10 bills is 10 1, the dollar value of the $1 and $10 bills must be equal. Therefore statement +1- gives us no new information, and we cannot find the number of $10 bills. +/- 'B::)A)?<T The problem states that the number of $1, $5, and $10 bills is in the ratio of 10 5 1, so letEs use an un;nown multiplier x to solve the problem. $1 bills $5 bills $10 bills Total <umber Ialue 10 x $10 x 5x $/5 x 1x $10 x 15 x $"5 x

Bsing x, we can see that there are 10x $1 bills with a value of $10x. :urthermore, there are 5x $5 bills with a value of $/5x. :inally, there are 1x $10 bills with a value of $10x. 'tatement +/- says that the total amount he has is $//5, so we can set up an equation as follows $10x 4 $/5x 4 $10x , $//5 $"5x , $//5 x,5 'ince there are 1x $10 bills this means that there are 5 $10 bills. The correct answer is .. /5. )t is tempting to view the information in the question as establishing a pattern as follows Kreen, >ellow, @ed, Kreen, >ellow, @ed, . . . Mowever, consider that the following nonGpattern is also possible Kreen, >ellow, @ed, Kreen, Kreen, Kreen, Kreen . . . )<'B::)A)?<T This tells us that the 1&th tile is Kreen or @ed but this tells us nothing about the /"th tile. 'tatement +1- alone is <=T sufficient. )<'B::)A)?<T This tells us that the 1#th tile is >ellow or @ed but this tells us nothing about the /"th tile. 'tatement +/- alone is <=T sufficient. (<$ +/- )<'B::)A)?<T Together, the statements yield the following possibilities for the 1&th and 1#th tiles K>, K@, @>, or @@ Mowever, only K> adheres to the rules given in the question. Thus, we ;now that tile 1& is green and tile 1# is yellow. Mowever, this does not help us to determine the color of the ne*t tile, much less tile /" +the one as;ed in the question-. :or e*ample, the next tile +tile /0- could be green or red. Thus, the statements ta;en together are still not sufficient. The correct answer is ?.

/!. ?ach bas;et must contain at least one of each type of fruit. We also must ensure that every bas;et contains less than twice as many apples as oranges. Therefore, the minimum number of apples that we need is equal to the number of bas;ets, since we can simply place one apple per bas;et +even if we had only 1 apple and 1 orange per bas;et, we would not be violating any conditions-. )f we are to divide the /0 oranges evenly, we ;now we will have 1, /, ", 5, 10, or /0 bas;ets +the factors of /0-. .ut because we donEt ;now the e*act number of bas;ets, we do not ;now how many apples we need. Thus, the question can be rephrased as PMow many bas;ets are thereQP )<'B::)A)?<T This tells us only that the number of bas;ets is even +halving an odd number of bas;ets would result in half of a bas;et-. 'ince we have /0 oranges that must be distributed evenly among an even number of bas;ets, we ;now we have /, ", 10, or /0 bas;ets. .ut because we still do not ;now e*actly how many bas;ets we have, we cannot ;now how many apples we will need. 'B::)A)?<T This tells us that 10 oranges +half of the original /0- would not be enough to place an orange in every bas;et. 'o we must have more than 10 bas;ets. 'ince we ;now the number of bas;ets is 1, /, ", 5, 10, or /0, we ;now that we must have /0 bas;ets. Therefore, we ;now how many apples we will need. The correct answer is .. 28. 2et the cost of each coat be *, the sales price be y. We Hust want to ;now what is /0+yG*-. :or 1, we ;new that /0+/yG*-,/"00, insufficient to find yG* :or /, we ;new that /0+y4/G*-,""0, we can get /0+yG*-,"00. )tEs sufficient. (nswer is . 29. :or 1, country ( can send # representatives, total number will #4&4!45454"1,!5N!5. (nswer is ? 30. 2et number of rows is a, number of the chairs in a row is b. 'o, bGa,1 :rom 1, ab!/, a&, b#, sufficient alone. :rom /, /bG1,1!, b,#, sufficient alone. (nswer is $. 31. 'tatement 1 is obviously insufficient 'tatement /, let :riday be *. To obtain the least value of *, the other five days should be, *G1, *G/, *G%, *G", *G5 'o, %&4*4*G14*G/4*G%4*G"4*G5,#0 5*,5! *,5!D5N11 %/.

2et attend fee be *, number of person be y :orm 1, +*G0.!5-+y4100-,*yGGGG100*G0.!5yG!5,0 :rom /, +*41.5-+yG100-,*y GGGG100*41.5yG150,0 Aombine 1 and /, we can get specific value of * and y. (nswer is A 33. Aombined 1 and /, three situations need to be studied GGGG2ast wee;4this wee;O%5, then *,+510G"&0-D/,15, the number of the items is "&0D15,%/ GGGG2ast wee;,%5, then *,"&0D%5,150D!. =r * can be resolved in the way *,+510G"&0-D+14%D/-,1/, two result are conflict. GGGG2ast wee;N,%5, then *,%0D+/8%D/-,10. The number of the items more than %5 ,+"&0G %5810-D/0,5, so, total number is %545,"/ (bove all, answer is ? 34. 1- is sufficient. /-. <o two members sold same number of tic;etsS the least numbers of the tic;ets they sold would be 0, 1, /. (nswer is $ 35. Pone ;ilogram of a certain coffee blend consists of V ;ilogram of type ) and > ;ilogram of type ))P means that V4>,1 Aombined A,5.5V4&.5>, we get V,+&.5GA-D/, >,+AG5.5-D/ Aombined AN,!.%, V,+&.5GA-D/O1./D/,0.5 (nswer is . 36. )t is somewhat tric;y. Bsually, we need two equations to solve two variables. :or e*ample, in this question, from 1, *,y,5, from /, /1*4/%y,1%0, the answer should be A. (ctually, the variables in such questions should be integers. Thus, hopefully, we can solve them with only one equation. /1*4/%y,1%0, we try *,1, /, %, ",5..and find that only *,", y,/ can fulfill the requirements. (nswer is .. To sum up, please be careful when you met such questions. 37. 3ore than 10 Laperbac; boo;s, at least is 11, and cost at least $&& :rom 1, 150D/5,5, at least 5 hardcover boo;s. :rom /, /50G150G&&,//, is not enough to buy a hardcover boo;. Aombined 1 and /, we ;now that 0uan bought 5 hardcover boo;s. (nswer is A

38. c,;*4t )n last month, cost,1000;4tS profit,1000+;450- G+1000;4t- ,50000Gt, so, we need to solve t. :rom 1, 150000,10008+;450-, there is no information about t. :rom /, +1000;4t-G+500;4t-,"5000, still cannot solve out t. (nswer is ? 39. +/45454"-D+500041/00041&000415000-850000 , /0 (nswer is . 40. The fine for one day $0.1 The fine two days $0./, as it is less than $0.14$0.% The fine for three days $0.", as it is less than $0./4$0.% The fine for four days $0."4$0.%,$0.!, as it is less than $0."8/ (nswer is . "1. 2et * be the height of the tree increase each year, then 7"45*G+"4"*-9D+"4"*- , 1D5 10* , "4 "* *, /D% "/. )n the origin plan, each one should pay VDT. (ctually, each of the remaining cowor;ers paid VD+TG'-. Then, VD+TG'- G VDT , '8V D T+TG'"%. The business produced a total of "* ra;es from <ovember through :ebruary. The storage situations were shown in the following table 'o, the total cost is 1"V80.1,1."V

3onth 3ar. (pr. 3ay 0une 0uly (ug. 'ept. =ct. Total 'torage !*D/ %* 5*D/ /* %*D/ * *D/ 0 1"*
"". In order to realize a profit, the company's revenue must be higher than the company's costs. We can
express this as an inequality using the information from the question:

If we distribute and move all terms to one side, we get:

We can factor this result:

When the value of p ma es this inequality true, we now we will have a profit. When the value of p does !"# ma e the inequality true, we will not have a profit. When p equals $ or %, the product is zero. &o the values of p that will ma e the inequality true 'i.e., will yield a negative product( must be either greater than %, less than $, or between $ and %. #o determine which is the case, we can test a sample value from each interval. If we try p ) *, we get:

&ince + is positive, we now that values of p greater than % will not ma e the inequality true and thus will not yield a profit. If we try p ) +, we get:

&ince + is positive, we now that the values of p less than $ will not ma e the inequality true and thus will not yield a profit. If we try p ) $.*, we get:

&ince ,.+* is negative, we now that values between $ and % will ma e the inequality true and will thus yield a profit. &ince p can be any positive value less than -.. 'we cannot have a negative price or a price of zero dollars(, there are -.. possible intervals between consecutive integer values of p. #he interval $ / p / % is 0ust one. #herefore, the probability that the company will realize a profit is -1-.. and the probability that it will !"# realize a profit is - , -1-.. or 221-... #he correct answer is 3.

"5. #o calculate the average daily deposit, we need to divide the sum of all the deposits up to and
including the given date by the number of days that have elapsed so far in the month. 4or example, if on 5une -$ the sum of all deposits to that date is 6+.,+$., then the average daily deposit to that date would be .

We are told that on a randomly chosen day in 5une the sum of all deposits to that day is a prime integer greater than -... We are then as ed to find the probability that the average daily deposit up to that day contains fewer than * decimal places. In order to answer this question, we need to consider how the numerator 'the sum of all deposits, which is defined as a prime integer greater than -..( interacts with the denominator 'a randomly selected date in 5une, which must therefore be some number between - and $.(. 4irst, are there certain denominators that 7 no matter the numerator 7 will always yield a quotient that contains fewer than * decimals8 9es. : fraction composed of any integer numerator and a denominator of - will always yield a quotient that contains fewer than * decimal places. #his ta es care of 5une 1. In addition, a fraction composed of any integer numerator and a denominator whose prime factorization contains only +s and1or *s will always yield a quotient that contains fewer than * decimal places. #his ta es cares of 5une 2, 4, 5, 8, 10, 16, 20, and 25. Why does this wor 8 ;onsider the chart below:

3enominator

:ny Integer divided by this denominator will yield either an integer quotient or < of 3ecimal a quotient ending in: =laces

.*

multiples of .+*

maximum of +

multiples of .+

>

multiples of .-+*

maximum of $

-.

multiples of .-

-?

multiples of ..?+*

maximum of %

+.

multiples of ..*

maximum of +

+*

multiples of ..%

maximum of +

What about the other dates in 5une8 If the chosen day is any other date, the denominator 'of the fraction that ma es up the average daily deposit( will contain prime factors other than + and1or * 'such as $ or @(. Aecall that the numerator 'of the fraction that ma es up the average daily deposit( is defined as a prime integer greater than -.. 'such as -.-(. #hus, the denominator will be composed of at least one prime factor 'other than + and1or *( that is not a factor of the numerator. #herefore, when the division ta es place, it will result in an infinite decimal. '#o understand this principle in greater detail read the explanatory note that follows this solution.( #herefore, of the $. days in 5une, only 2 '5une -, +, %, *, >, -., -?, +., and +*( will produce an average daily deposit that contains fewer than * decimal places: #he correct answer is 3. .

Explanatory Note: Why will an infinite decimal result whenever a numerator is divided by a denominator composed of prime factors 'other than + and1or *( that are not factors of the numerator8 ;onsider division as a process that ends when a remainder of . is reached. Bet's loo at - 'the numerator( divided by @ 'the denominator(, for example. If you divide - by @ on your calculator, you will see that it equals .-%+>... #his decimal will go on infinitely because @ will never divide evenly into the remainder. #hat is, a remainder of . will never be reached.

, and so on... 4or contrast, let's loo at +$ divided by *:

&o +$1* ) %.?. When the first remainder is divided by *, the division will end because the first remainder '$( is treated as if it were a multiple of -. to facilitate the division and * divides evenly into multiples of -.. Cy the same to en, the remainder when an odd number is divided by + is always -, which is treated as if it were -. to facilitate the division. -. divided by + is * 'hence the .*( with no remainder. When dividing by primes that are not factors of -. 'e.g., $, @, --, etc.(, however, the process continues infinitely because the remainders will always be treated as if they were multiples of -. but the primes cannot divide cleanly into -., thus creating an endless series of remainders to be divided. If the divisor contains +'s and1or *'s in addition to other prime factors, the infinite decimal created by the other prime factors will be divided by the +'s and1or *'s but will still be infinite.

%?. It might be tempting to thin that either statement is sufficient to answer this question. :fter all, pouring water from the larger container to the smaller container will leave exactly + gallons of water in the larger container. Aepeating this operation twice will yield % gallons of water. #he problem is , where would these % gallons of water accumulate8 We will need to use one of the containers. Dowever, neither statement alone tells us whether one of the containers will hold % gallons of water. "n the other hand, statements '-( and '+( ta en together ensure that the first container can hold at least % gallons of water. We now this because 'from statement -( the first container holds + gallons more than the second container, which 'from statement +( holds + gallons more than the third container, which must have a capacity greater than .. &ince we now that the first container has a capacity of at least % gallons, there are several ways of measuring out this exact amount. "ne method is as follows: ;ompletely fill the first container with water. #hen pour out 0ust enough water from the first container to fill the third container to the brim. !ow, % gallons of water remain in the first container. :lternatively: 4ill the first container to the brim. =our out 0ust enough water from the first container to fill the second container to the brim. #here are now + gallons of water in the first container. !ow pour water from the second container to fill the third container to the brim. #here are now + gallons of water in the second container. 4inally, pour all the water from the second container into the first container. #here are now % gallons of water in the first container. %@. We can answer this by eeping trac of how many cubes are lopped off of each side as the cube is trimmed '-. x -. E -. x 2 E 2 x 2 E ...(, but this approach is tedious and error prone. : more efficient method is to determine the final dimensions of the trimmed cube, then find the difference between the dimensions of the trimmed and original cubes. Bet's call the first face :, second face C, and third face ;. Cy the end of the operation, we will have removed + layers each from faces C and ;, and $ layers from face :. &o C now is > cubes long, ; is > cubes long, and : is @ cubes long. #he resulting solid has dimensions > x > x @ cubes or %%> cubes. We began with -... cubes, so -... , %%> ) **+. #hus, **+ cubes have been removed. #he correct answer is C. %>. In order to determine the length of the line, we need to now how many people are standing in it. #hus, rephrase the question as follows: Dow many people are standing in the line8 &tatement '-( says that there are three people in front of ;handra and three people behind Fen. ;onsider the following different scenarios: #he line might loo li e this: 'Cac ( G G G Fen G ;handra G G G '4ront( "A #he line might loo s li e this: 'Cac ( ;handra G G Fen '4ront( #he number of people in the line depends on several factors, including whether ;handra is in front of Fen and how many people are standing between ;handra and Fen. &ince there are many different scenarios, statement '-( is not sufficient to answer the question.

&tatement '+( says that two people are standing between ;handra and Fen. Dere, we don't now how many people are ahead or behind Fen and ;handra. &ince there are many different scenarios, statement '+( is not sufficient to answer the question. #a ing both statements together, we still don't now whether ;handra is in front of Fen or vice versa, and therefore we still have two different possibilities: #he line might loo li e this: 'Cac ( G G G Fen G G ;handra G G G '4ront( "A #he line might loo li e this: 'Cac ( ;handra G G Fen '4ront( #herefore, the correct answer is 'H(: &tatements '-( and '+( #"IH#DHA are !"# sufficient. %2. Cegin by rephrasing, or simplifying, the original question. &ince the rules of the game involve the negative of the sum of two dice, one way of restating this problem is that whoever gets the higher sum B"&H& the game. #hin ing about the sum of the two dice is easier than thin ing about the negative of the sum of the two dice. #hus, let's rephrase the question as: Who lost the game8 'Fnowing this will obviously allow us to answer the original question, who won the game.( &tatement '-( gives us information about the first of !ina's dice, but it does not tell us anything about the second. ;onsider the following two possibilities:

!ina's 4irst Aoll !ina's &econd Aoll ;:&H "!H ;:&H #W" 3 3 * 7*

#eri's &um 1 1

Digher &um ) Boser !ina #eri

!otice that in both cases, !ina's first roll is greater than #eri's &um. Dowever, in ;ase "ne !ina loses, but in ;ase #wo #eri loses. #hus, this information is not sufficient to answer the question. &tatement '+( gives us information about the second of !ina's dice, but it does not tell us anything about the first. Jsing the same logic as for the previous statement, this is not sufficient on its own to answer the question. ;ombining the information contained in both statements, one may be tempted to conclude that !ina's sum must be higher than #eri's sum. Dowever, one must test scenarios involving both positive and negative rolls. ;onsider the following two possibilities.

!ina's 4irst Aoll !ina's &econd Aoll ;:&H "!H ;:&H #W" 3 3 4 4

#eri's &um 5 5

Digher &um ) Boser !ina #eri

!otice that in both cases, !ina's first roll is greater than #eri's &um and !ina's second roll is greater than #eri's sum. Dowever, in ;ase "ne !ina loses, but in ;ase #wo #eri loses. #hus, this information is not sufficient to answer the question. #he correct answer is H: &tatements '-( and '+( #"IH#DHA are !"# sufficient. *.. Hvery third :lb gives a clic . #his means no clic is awarded until the third :lb is captured. #he second clic is not awarded until the sixth :lb is captured. &imilarly, a tic is not awarded until the fourth Cer is captured. We are told that the product clic s x tic s ) @@. #hus, there are four possibilities: - K @@, @ K --, -- K @, @@ K -. ;lic s :warded :lbs ;aptured $, % or * #ic s :warded Cer s ;aptured @@ $.>, $.2, $-., or $--

@ -@@

+-, ++, or +$ $$, $%, or $*

-@

%%, %*, %? or %@ +>, +2, $. or $%, *, ?, or @

+$-, +$+ or +$$ -

&tatement '-( tells us that the difference between :lbs captured and Cer s captured is @. Boo ing at the chart, the only way to get a difference of @ between :lbs captured and Cer s captured is with $* :lbs and +> Cer s. #herefore, statement '-( is sufficient to answer the question,,there must have been $* :lbs captured. &tatement '+( says the number of :lbs captured is divisible by %. :gain, loo ing at the chart, we see that the number of :lbs captured must be % or +$+. #herefore, statement '+( is not sufficient to answer the question,,we do not now how many :lbs were captured. #he correct answer is :: &tatement '-( alone is sufficient, but statement '+( alone is not sufficient.

51. The question gives a function with two unknown constants and two data points. In order to solve for the position of the object after 4 seconds, we need to first solve for the contants r and b. We can do this b creating two equations fro! the two data points given" p#$% & 41 & r#$% ' 5#$%$ ( b 41 & $r ' $) ( b *1 & $r ( b p#5% & $* & r#5% ' 5#5%$ ( b $* & 5r ' 1$5 ( b 151 & 5r ( b We can now solve these equations for r and b using substitution" *1 & $r ( b #*1 ' $r% & b 151 & 5r ( b 151 & 5r ( #*1 ' $r% 151 & +r ( *1 ,) & +r r & +) -ubstituting back in, we can find b" *1 & $r ( b *1 & $#+)% ( b b&1 -o, we can rewrite the original function and plug in t & 4 to find our answer" p#t% & +)t ' 5t$ ( 1 p#4% & +)#4% ' 5#4%$ ( 1 p#4% & 1$) ' .) ( 1 p#4% & 41 The correct answer is /. 5$. 0et us call the Trussian1s current age a. Therefore the Trussian1s current weight is .

-eventeen ears after he is twice as old as he is now, the Trussian1s age will be weight will therefore be less than his future weight, follows" .We are told that the Trussian1s current weight, . Therefore,

and his , is three keils

.We can solve the equation as

a & 1* or 4. 2owever, we are told that the Trussian is a teenager so he !ust be 1* ears old. The correct answer is 3. 5+. This proble! is easier to think about with real values. 0et1s assu!e that there are $ high level officials. This !eans that each of these $ high level officials supervises 4 #or x$% !id4level officials, and that each of these 4 !id4level officials supervises . #or x+% low4level officials. It is possible that the supervisors do not share an subordinates. If this is the case, then, given $ high level officials, there !ust be $#4% & . !id4level officials, and .#.% & *4 low4level officials. 5lternativel , it is possible that the supervisors share all or so!e subordinates. In other words, given $ high level officials, it is possible that there are as few as 4 !id4level officials #as each of the $ high4 level officials supervise the sa!e 4 !id4level officials% and as few as . low4level officials #as each of the 4 !id4level officials supervise the sa!e . low4level officials%. -tate!ent #1% tells us that there are fewer than *) low4level officials. This alone does not allow us to deter!ine how !an high4level officials there are. 6or e7a!ple, there !ight be $ high level officials, who each supervise the sa!e 4 !id4level officials, who, in turn, each supervise the sa!e . low4level officials. 5lternativel , there !ight be + high4level officials, who each supervise the sa!e , !id4level officials, who, in turn, each supervise the sa!e $8 low4level officials. -tate!ent #$% tells us that no official is supervised b !ore than one person, which !eans that supervisors do not share an subordinates. 5lone, this does not tell us an thing about the nu!ber of high4level officials. 3o!bining state!ents 1 and $, we can test out different possibilities. If x & 1, there is 1 high4level official, who supervises 1 !id4level official #1$ & 1%, who, in turn, supervises 1 low4level official #1+ & 1%. If x & $, there are $ high4level officials, who each supervise a unique group of 4 !id4level officials, ielding . !id4level officials in total. 9ach of these . !id4level officials supervise a unique group of . low4level officials, ielding *4 low4level officials in total. 2owever, this cannot be the case since we are told that there are fewer than *) low4level officials. Therefore, based on both state!ents taken together, there !ust be onl 1 high4level official. The correct answer is 3" :;T2 state!ents T;<9T29= are sufficient, but >9IT29= state!ent 50;>9 is sufficient. 54.

?se algebra to solve this proble! as follows" 0et the x & the nu!ber of donuts @i! originall ordered. -ince he paid A15 for these donuts, the price per donut for his original order is A15B7. When he leaves, @i! receives + free donuts changing the price per donut to A15B#7 ( +%. In addition, we know that the price per doCen donuts was A$ per doCen cheaper when he leaves, equivalent to a per donut savings of A$B1$ & 1B* dollars. ?sing this infor!ation, we can set up an equation that states that the original price per donut less 1B* of a dollar is equal to the price per donut after the addition of + donuts"

We can now solve for x as follows"

The onl positive solution of 7 is 15. 2ence, @i! left the donut shop with 7 ( + & 1. donuts. The correct answer is 5.

55. In questions like this, it helps to record the given infor!ation in a table. ?pon initial reading, the second sentence is probabl ver confusing but what is clear is that it discusses the ages of the two bo s at two different points in ti!e" let1s refer to the! as DnowE, and DthenE. -o, let1s construct a table such as the one below. 0et x and y denote the bo s1 ages DnowE" @ohnn 1s age :obb 1s age >ow x y then >ow, re4read the first few words of the second sentence" D@ohnn 1s age now is the sa!e as :obb 1s age . . . FthenGE. We can fill in one !ore entr of the table as shown" @ohnn 1s age :obb 1s age >ow x y then x 6inall , the rest of the second sentence tells us that DthenE was the ti!e when @ohnn 1s age was half :obb 1s current ageH i.e., @ohnn 1s age DthenE was #1B$% y. We can co!plete the table as follows" @ohnn 1s age :obb 1s age >ow x y then #1B$%y x ;ne wa to solve this proble! is to realiCe that, as two people age, the ratio of their ages changes but the difference in their ages re!ains constant. In particular, the difference in the bo s ages Dnow1E !ust be the sa!e as the difference in their ages DthenE. This leads to the equation" y 4 x & x 4 #1B$%y, which reduces to x & #+B4%yH @ohnn is currentl three4fourths as old as :obb . Without another equation, however, we can1t solve for the values of either x or y. #5lternativel , we could co!pute the elapsed ti!e between DthenE and DnowE for each bo and set the two equalH this leads to the sa!e equation as above.%

#1% I>-?66I3I9>T" :obb 1s age at the ti!e of @ohnn 1s birth is the sa!e as the difference between their ages, y 4 x. -o state!ent #1% tells us that y & 4#y 4 x%, which reduces to x & #+B4%y. This adds no !ore infor!ation to what we alread knewI -tate!ent #1% is insufficient. #$% -?66I3I9>T" This tells us that :obb is * ears older than @ohnn H i.e., y & x ( *. This gives us a second equations in the two unknowns so, e7cept in so!e rare cases, we should be able to solve for both 7 and 44 state!ent #$% is sufficient. @ust to verif , substitute x & #+B4%y into the second equation to obtain y & #+B4%y ( * , which i!plies y & $4. :obb is currentl $4 and @ohnn is currentl 1.. The correct answer is :, -tate!ent #$% 50;>9 is sufficient to answer the question, but state!ent #1% alone is not. 56. 6irst, let c be the nu!ber of cash!ere blaCers produced in an given week and let m be the nu!ber of !ohair blaCers produced in an given week. 0et p be the total profit on blaCers for an given week. -ince the profit on cash!ere blaCers is A4) per blaCer and the profit on !ohair blaCers is A+5 per blaCer, we can construct the equation p = 4)c + +5m. In order to know the !a7i!u! potential value of p, we need to know the !a7i!u! values of c and m. -tate!ent #1% tells us that the !a7i!u! nu!ber of cutting hours per week is $)) and that the !a7i!u! nu!ber of sewing hours per week is $)). -ince it takes 4 hours of cutting to produce a cash!ere blaCer and 4 hours of cutting to produce a !ohair blaCer, we can construct the following inequalit " . -ince it takes * hours of sewing to produce a cash!ere blaCer and $ hours of sewing to produce a !ohair blaCer, we can construct the following inequalit " . In order to !a7i!iCe the nu!ber of blaCers produced, the co!pan should use all available cutting and sewing ti!e. -o we can construct the following equations"

-ince both equations equal $)), we can set the! equal to each other and solve"

-o when m & $5 and c = $5, all available cutting and sewing ti!e will be used. If p = 4)c ( +5m, the profit in this scenario will be 4)#$5% ( +5#$5% or A1,.85. Is this the !a7i!u! potential profitJ -ince the profit !argin on cash!ere is higher, !ight it be possible that producing onl cash!ere blaCers would be !ore profitable than producing both t pesJ If no !ohair blaCers are !ade, then the largest nu!ber of cash!ere blaCers that could be !ade will be the value of c that satisfies *c & $)) #re!e!ber, it takes * hours of sewing to !ake a cash!ere blaCer%. -o c could have a !a7i!u! value of ++ #the co!pan cannot sell 1B+ of a blaCer%. -o producing onl cash!ere blaCers would net a potential profit of 4)#++% or A1,+$). This is less than A1,.85, so it would not !a7i!iCe profit. -ince !ohair blaCers take less ti!e to produce, perhaps producing onl !ohair blaCers would ield a higher profit. If no cash!ere blaCers are produced, then the largest nu!ber of !ohair blaCers that could be !ade will be the value of m that satisfies 4m & $)) #re!e!ber, it takes 4 hours of cutting to

produce a !ohair blaCer%. -o m would have a !a7i!u! value of 5) in this scenario and the profit would be +5#5)% or A1,85). This is less than A1,.85, so it would not !a7i!iCe profit. -o producing onl one t pe of blaCer will not !a7i!iCe potential profit, and producing both t pes of blaCer !a7i!iCes potential profit when m and c both equal $5. -tate!ent #1% is sufficient. -tate!ent #$% tells us that the wholesale cost of cash!ere cloth is twice that of !ohair cloth. This infor!ation is irrelevant because the cost of the !aterials is alread taken into account b the profit !argins of A4) and A+5 given in the question ste!. -tate!ent #$% is insufficient. The answer is 5" -tate!ent #1% alone is sufficient, but state!ent #$% alone is not. 57. 9ach ear, the age of the bo increases b 1. 9ach ear, the su! of the ages of the two girls increases b $ #as each girl gets older b one ear, and there are two of the!%. 0et1s sa that the age of the bo toda is equal to x, while the co!bined ages of the girls toda is equal to y. Then, ne7t ear the figures will be x ( 1 and y ( $, respectivel . The proble! states that these two figures will be equal, which ields the following equation" x ( 1 & y ( $ which can be si!plified to x = y + 1 #This is consistent with the fact that the su! of the ages of the two girls toda is s!aller than the age of the bo toda .% Three ears fro! now, the co!bined age of the girls will be y ( +#$% & y ( *. Three ears fro! now, the bo 1s age will be x ( +. ?sing the fact #fro! above% that x & y ( 1, the bo 1s age three ears fro! now can be written as x ( + & #y ( 1% ( + & y ( 4. The proble! asks for the difference between the age of the bo three ears fro! toda and the co!bined ages of the girls three ears fro! toda . This difference equals y ( 4 ' #y ( *% & '$. The correct answer is /. Klug in real nu!bers to see if this !akes sense. 0et the girls be 4 and * in age. The su! of their ages toda is 1). The bo 1s age toda is then #1) ( 1% & 11. Three ears fro! toda , the girls will be 8 and , respectivel , so their co!bined age will be 1*. Three ears fro! toda , the bo will be 14. :e careful" The question asks for the difference between the bo 1s age and the su! of the girls ages three ears fro! toda . Which one will be oungerJ The bo . -o the difference between the bo 1s age and the co!bined age of the girls will be a negative value" 14 ' 1* & ' $. 58. ?se a chart to keep track of the ages in this proble!" x ears ago 3ore Tania C4x T4x >;W C T in x ears C(x T(x

Then write algebraic e7pressions to represent the infor!ation given in the proble!" x ears ago, 3or was one fifth as old as Tania 5#C ' x% & T ' x 5C ' 5x & T ' x 5C ' T & 4x In 7 ears, Tania will be twice as old as 3or $#C ( x% & T ( x $C ( $x & T ( x T ' $C & x

-ubstitute T 4 $C in for x in the first equation and solve"

5C 4 T & 4#T 4 $C% 5C 4 T & 4T 4 .C 1+C & 5T

The correct answer is 3. 5,. The question does not ask for the actual nu!ber of ears ago that ani!al z beca!e e7tinct. Instead it asks for t, the nu!ber of ears scientists predicted it would take for ani!al z to beco!e e7tinct. #1% I>-?66I3I9>T" This tells us that ani!al z beca!e e7tinct 4 ears ago but it does not provide infor!ation about t. #$% I>-?66I3I9>T" This provides a relationship between the predicted ti!e of e7tinction ti!e and the actual ti!e of e7tinction but does not provide an actual values for either. #1% 5>/ #$% I>-?66I3I9>T" The easiest wa to approach this proble! is to i!agine a ti!e line fro! ) to 1). The scientists !ade their prediction 1) ears ago, or at ) ears. 6ro! state!ent #1% we know that ani!al z beca!e e7tinct 4 ears ago, or at * ears. 6ro! state!ent #$% we know that if the scientists had e7tended their prediction b + ears the would have been incorrect b $ ears. The ke to this question is to realiCe that Lincorrect b $ earsL could !ean $ ears in either direction" * ( $ & . ears or * ' $ & 4 ears. 6ro! here, we can write two si!ple equations" t(+&. t&5 ;= t(+&4 t&1

This gives us two different values for t, which !eans that #1% and #$% together are not sufficient to co!e up with one definitive value for t. The correct answer is 9.

50. To answer this question, we need to minimiJe the value of l , +!.5 C x-" 4 &.#!1.05. 'ince we do not need to determine the actual minimum longevity, we do not need to find the value of the second component in our formula, &.#!1.05, which will remain constant for any level of x. Therefore, to minimiJe longevity, we need to minimiJe the value of the first component in our formula, i.e. +!.5 C x-". 'ince we are raising the e*pression +!.5 C x- to an even e*ponent, ", the value of +!.5 C x-" will always be nonGnegative, i.e. positive or Jero. Thus, to minimiJe this outcome, we need to find the value of x for which +!.5 C x-" , 0. +!.5 C x-" , 0 !.5 C x , 0 x , !.5 Therefore, the metal construction will have minimal longevity for the value of x , !.5, i.e. when the density of the underlying material will be equal to !.5 gDcm%. The correct answer is A.

Calculations, Exponents, Basic Algebra 1. The first culprit in this e7pression is the radical in the deno!inator. =adicals in the deno!inator are dealt with b !ultipl ing the fraction with an e7pression that is equal to 1 but contains a Dcanceling radicalE in both the nu!erator and deno!inator.

6or e7a!ple,

is si!plified b !ultipl ing b

. , we !ultipl b the conjugate, as follows"

In the case of a co!ple7 radical, such as

-i!plif ing radicals in the deno!inator with conjugate radical e7pressions is ver useful on challenging <M5T radical questions. The correct answer is 3. 2. The <M5T does not require ou to know how to evaluate an integral root of an general integer, but ou are e7pected to understand how to evaluate an integer raised to an integer power. 2ence, ou should i!!ediatel realiCe that there !ust be a wa we can transfor! each of the e7pressions into an e7pression that we can evaluate. The !ost obvious wa to transfor! a root into a power is to raise it to a higher power. -ince we are tr ing to co!pare the e7pressions, a reasonable transfor!ation is to raise each of the e7pressions to the sa!e power. What power should we useJ

-ince , in order to get integral powers of x #i.e., nBm is an integer% we should raise all of the roots to the 0east 3o!!on Multiple of the ms . We have roots of +, *, 1), and 15, so the 03M is equal to +). Therefore, we should raise each of the e7pressions to the +) th power as follows"

Thus, the original e7pressions in increasing order are as follows" . 3. In order to rid the e7pression of square roots, let1s first square the entire e7pression. We are allowed to do this as long as we re!e!ber to LunsquareL whatever solution we get at that end.

>otice that the new e7pression is of the for! . =ecall that =eturning to our e7pression" , while >otice that In order to si!plif Thus,

where

. This is one of the <M5T1s favorite e7pressions.

and

. e7pression left to si!plif . . . . 5nd recall that

neatl si!plifies to 4.. This leaves onl the , recall that

>otice that the e7pression under the square root sign is of the for! . This is another one of the <M5T1s favorite e7pressions. =eturning to our e7pression"

. 6inall then" . :ut now we !ust re!e!ber to LunsquareL #or take the square root of% our answer" . Therefore, the correct answer is /. 4. We can si!plif the question as follows" .a#1B4%b & J N:reak all non4pri!es down to pri!es.O #$+%a#$4$%b & J NMultipl e7ponents taken on the sa!e base.O #$+a%#$4$b% & J N5dd e7ponents since the two bases are equal.O $+a 4 $b & J We can rephrase the question as Lwhat is +a 4$bJL #1% -?66I3I9>T" b & 1.5a, so $b & +a. This !eans that +a 4 $b & ). #$% I>-?66I3I9>T" This state!ent gives us no infor!ation about b. The correct answer is 5 5.

The distance fro! G to H is 51+ 4 51$. The distance between and two consecutive points is constant, so the distance fro! 5 to < will be * ti!es the distance fro! G to H or *#51+ ' 51$%. The value of A, therefore, will be equal to the value of G !inus the distance fro! A to G" 51$ ' *#51+ ' 51$% 51$#1 ' $4% The correct answer is :. 51$ ' *N51$#5 ' 1%O #4$+%51$. 51$ ' *#51$%#4%

*. #+5x ( +5x + +5x%#45x + 45x ( 45x ( 45x% & 5x + #1 ( 1 ( 1% P 45x#1 ( 1 ( 1 ( 1% & +#+5x% P 4#45x% & +5x(1 P 45x(1 & #+ P 4%5x(1 & 1$5x(1 =e!e!ber that when ou !ultipl different bases raised to the -5M9 e7ponent, the product is si!pl the product of the bases raised to their co!!on e7ponent. The correct answer is 5. 8. 6irst, let us si!plif the e7ponential equation" #$a%#+b%#5c% & 1$#$k%#+l%#5m% #$a%#+b%#5c% & #+%#4%#$k%#+l%#5m% #$a%#+b%#5c% & #+1%#$$%#$k%#+l%#5m% #$a%#+b%#5c% & #$k+2%#+l ( 1%#5m% When the bases on both sides of an equation are equal and the bases are pri!e nu!bers, the e7ponents of the respective bases !ust also be equal" a & k ( $H b & l ( 1H and c & m. >ow recall that a, b, and c represent the hundreds, tens, and units digits of the three4digit integer xH si!ilarl , k, l, and m represent the hundreds, tens, and units digits of the three4digit integer y. Therefore, the hundreds digit of x is $ greater than the hundreds digit of yH the tens digit of x is 1 greater than the tens digit of yH finall , the units digit of x is equal to the units digit of y. ?sing this infor!ation, we can set up our subtraction proble! and find the value of # x ' y%" abc klm $1) The correct answer is 3. .. #1% -?66I3I9>T" -tate!ent#1% tells us that x Q $+4, so we want to prove that $+4 Q 1)1). We1ll prove this b !anipulating the e7pression $+4. $+4 & #$4%#$+)% $+4 & 1*#$1)%+ >ow $1) & 1)$4, and 1)$4 is greater than 1)+. Therefore" $+4 Q 1*#1)+%+ $+4 Q 1*#1),% $+4 Q 1.*#1)1)%. -ince $+4 Q 1.*#1)1)% and 1.*#1)1)% Q 1)1), then $+4 Q 1)1). #$% -?66I3I9>T" -tate!ent #$% tells us that that x & $+5, so we need to deter!ine if $+5 Q 1)1). -tate!ent #1% showed that $+4 Q 1)1), therefore $+5 Q 1)1). The correct answer is /. ,. 5 radical e7pression in a deno!inator is considered non4standard. To eli!inate a radical in the deno!inator, we can !ultipl both the nu!erator and the deno!inator b the conjugate of that deno!inator.

The correct answer is 3 1). 6irst rewrite the e7pression in the question using onl pri!e bases #4 is not pri!e%, as follows" $a$$b.

#1% -?66I3I9>T" We can substitute 4$b for a into the e7pression in the question. What is the value of #$4$b%#$$b%J This can be si!plified to $4$b($b & $) & 1. #$% I>-?66I3I9>T" We have no infor!ation about the value of a. The correct answer is 5. 11. 5n effective strateg for proble!s involving e7ponents is to break the bases of all the e7ponents into pri!e factors. This technique will allow us to co!bine like ter!s" $84x ( $ P 1*$4$x P +*x P ,* ' $x & 1 #++%4x ( $ P #$ P +4%4$x P #$$ P +$%x P #+$%* ' $x & 1 +1$x ( * P $4$x P +4.x P $$x P +$x P +1$ ' 4x & 1 $4$x ( $x P +1$x ( * ' .x ( $x ( 1$ ' 4x & 1 $) P + $x ( 1. & 1 + $x ( 1. & 1 + $x ( 1. & +) $x ( 1. & ) $x & 41. x & 4, The correct answer is 5. 1$. 0et1s rewrite the right side of the equation in base $ and base +" #$$x(1%#+$y41% & #$+%x#++%y. This can be rewritten as" #$$x(1%#+$y41% & $+x++y -ince both bases on either side of the equation are pri!e, we can set the e7ponents of each respective base equal to one another" $x ( 1 & +x, so x & 1 $y ' 1 & +y, so y & 41 Therefore, x ( y & 1 ( #41% & ). The correct answer is 3 1+. :efore dealing with this e7pression, it is helpful to re!e!ber several general e7ponent rules" When !ultipl ing e7pressions with the sa!e base, 5// the e7ponents first" #+$%#++% & #+%#+%#+%#+%#+% & +5 & +$ ( +

When dividing e7pressions with the sa!e base, -?:T=53T the e7ponents first" #+5%B#+$% & #+%#+%#+%#+%#+% B #+%#+% & #+%#+%#+% & ++ & +5 4 $ When raising a power to a power, co!bine e7ponents b M?0TIK0RI>< the!" #+$%4 & #+$%#+$%#+$%#+$% & #+ P +%#+ P +%#+ P +%#+ P +% & #+%#+%#+%#+%#+%#+%#+%#+% & +. & +$#4% In this question, we are asked to solve an e7pression with !an e7ponents, but none of the! have co!!on bases, at least not as currentl written. 2owever, so!e of the bases have pri!e factors in co!!on. 0ook at each part of the e7pression and break each into its factored for!. If we look at the ter!s in the nu!erator on the left side" *$ & #$ P +%$ & $$ P +$ 44 & $ P $ P 11 & $$ P 111 5x cannot be factored $) & $ P $ P 5 & $$ P 51 >ote that several of the ter!s in the nu!erator have bases in co!!on, so the nu!erator si!plifies b 5//I>< the e7ponents of those ter!s" #$$ P +$%#$$ P 111%#5x%#$$ P 51% & #$*%#+$%#5#x ( 1%%#111% If we look at the ter!s in the deno!inator on the left side" .$ & #$ P $ P $%$ & #$+%$ & $* , & + P + & +$ ;n the right side of the equation" 1+85 & 5 P 5 P 5 P 11 & 5+ P 111 >ow that each e7ponent and large nu!ber is e7pressed in ter!s of its pri!e factors, we can put the equation back together. Then we1ll see what can cancel to si!plif the entire equation" #$*%#+$%#5#x ( 1%%#111% S #$*%#+$% & 5+ P 111 3anceling the $* and +$ that appear in the nu!erator and deno!inator of the left side, and canceling the 11 that appears on each side of the equal sign" 5#x ( 1% & 5+ x(1&+ x&$ The correct answer is /. 14. If we know y, we can solve for x. Thus, we can rephrase the question, LWhat is yJL #1% -?66I3I9>T" If y$ & *$5, we know that y & $5 or 4$5. 2owever, since 5x will be positive no !atter what x is, and 5x & y, then y !ust be positive. Thus, y !ust be $5. If y is $5, we know that x & $. #$% -?66I3I9>T" If y+ & 15,*$5, y !ust equal $5. #Tip" In order to understand a nu!ber like 15,*$5 e7ponentiall , tr breaking it down into its roots. 15,*$5 & 5 P 5 P 5 P 5 P 5 P 5 & $5 P $5 P $5 & $5 +.% If y is $5, we know that x & $. The correct answer is /. 15. This question !ight be rephrased D2ow !an golf balls do Wend and Kedro have co!binedJE ;therwise, we !ust si!pl find the nu!ber of balls possessed b @i!. #1% I>-?66I3I9>T" ;bserve that @i! could have $ balls and Wend *, or @i! could have + balls while

Wend has ,. #$% I>-?66I3I9>T" If Kedro has 1B$ of the golf balls, Kedro has 1$ balls. 2owever, this state!ent gives no infor!ation about the nu!ber of balls possessed b @i! or Wend . #1% 5>/ #$% -?66I3I9>T" -tate!ent #$% tells us that Kedro has 1$ balls. Therefore, Wend and @i! collectivel have the re!aining 1$ balls. -tate!ent #1% tells us that @i! has 1B+ of the nu!ber of Wend Gs golf balls. 0et j & the nu!ber of @i!Gs golf balls and w & the nu!ber of Wend Gs golf balls. j & wB+ Multipl ing both sides b + ields +j & w <iven that j ( w & 1$, we can substitute that j ( +j & 1$. If 4j & 1$, j & +. The correct answer is 3. 1*. The best wa to answer this question is to use the e7ponential rules to si!plif the question ste!, then anal Ce each state!ent based on the si!plified equation. #+$8%#+51)%#z% & #5.%#81)%#,14%#xy% :reak up the +51) and si!plif the ,14 #+$8%#51)%#81)%#z% & #5.%#81)%#+$.%#xy% /ivide both sides b co!!on ter!s 5., 81), +$8 #5$%#z% & +xy #1% -?66I3I9>T" 5nal Cing the si!plified equation above, we can conclude that z !ust have a factor of + to balance the + on the right side of the equation. -tate!ent #1% sa s that z is pri!e, so z cannot have another factor besides the +. Therefore z & +. -ince z & +, the left side of the equation is 85, so xy & $5. The onl integers greater than 1 that satisf this equation are x & 5 and y & $, so state!ent #1% is sufficient. Kut differentl , the e7pression xy !ust provide the two fives that we have on the left side of the equation. The onl wa to get two fives if x and y are integers greater than 1 is if x & 5 and y & $. #$% -?66I3I9>T" 5nal Cing the si!plified equation above, we can conclude that x !ust have a factor of 5 to balance out the 5$ on the left side. -ince state!ent #$% sa s that x is pri!e, x cannot have an other factors, so x & 5. Therefore state!ent #$% is sufficient. The correct answer is /. 18. It is te!pting to e7press both sides of the equation 4 4x & 1*)) as powers of 4 and to tr and solve for x. 2owever, if we do that, we get a power of five on the right side as well" 44x & 1* P 1)) 44x & 4$ P 4 P $5 44x & 4+ P 5$ It beco!es clear that x is not an integer and that we canGt solve the question this wa . 0etGs tr !anipulating the e7pression about which we are being asked. #4x1%$ & 4$x $ If we further si!plif we get the e7pression 4$xB4$ To solve this e7pression, all we need is to find the value of 4$x >ow letGs look back at our original equation. If 4 4x & 1*)), we can find the value of 4$x b taking the square root of both sides of the equation. Taking the square root of an e7ponential e7pression is tanta!ount to halving its e7ponent.

-ince the question asks for 4$xB4$, the answer is 4)B4$, which si!plifies to 4)B1* or 5B$. The correct answer is / 1.. #15x ( 15x(1% & 15y4y N157 ( 15x#151%O & 15y4y #15x %#1 ( 15% & 15y4y #15x%#1*% & 15y4y x #+ %#5x%#$4% & #+y%#5y%#$$y% -ince both sides of the equation are broken down to the product of pri!e bases, the respective e7ponents of like bases !ust be equal. $y & 4 so y & $. x & y so x & $. The correct answer is 5 1,. In proble!s that involve e7ponential e7pressions on both sides of the equation, it is i!perative to rewrite the bases so that either the sa!e base or the sa!e e7ponent appears on both sides of the equation. 2ere, we can get a co!!on base b replacing the , with + $. +m+m+m & ,n +m+m+m & #+$%n ++m = +$n -ince the bases are the sa!e, the e7ponents !ust be equal. +m & $ n m/n & $B+ The correct answer is : 5). #1% -?66I3I9>T" We can rewrite this equation in a base of +" +b ( $ & +5, which !eans that b ( $ & 5 and therefore b & +. We can plug this value into the equation a & +b ' 1 to solve for a. #$% -?66I3I9>T" We can set the right side of this equation equal to the right side of the equation in the quesiton #both sides equal a%. +b ' 1 & +$b ' 4, which !eans that b ' 1 & $b ' 4 and therefore b & +. We can plug this value into the equation a & +b ' 1 to solve for a. The correct answer is / $1. =ecogniCe here the basic for! #x 'y%$, which equals x$ 4 $xy ( y$. corresponds here to x, and -o the e7pression can be si!plified to" corresponds to y.

?nder the radical, recogniCe the basic for! #a ( b%#a ' b%, which equals a$ ' b$. The e7pression can be further si!plified to"

The correct answer is 3 $$. The ke to this question is to recogniCe the co!!on algebraic identit " a$ ' b$ & #a ( b%#a ' b% In this question, the a ter! is xB+ and the b ter! is $By, which !akes the identit fro! the question equal to" x$B, ' 4By$ & #xB+ ' $By%#xB+ ( $By% & 1$ #1% -?66I3I9>T" -ubstituting the infor!ation fro! this state!ent into the equation fro! the question" #xB+ ' $By%#xB+ ( $By% & 1$ #xB+ ' $By%#*% & 1$ xB+ ' $By & $ We now have two equations with x and y" xB+ ( $By & * #fro! this state!ent% xB+ ' $By & $ #fro! the substitution above% 3o!bine the two equations #b adding%, then si!plif " #$%#xB+% & . #the y ter!s cancelled% xB+ & 4 x & 1$ #$% -?66I3I9>T" -ubstituting the infor!ation fro! this state!ent into the equation fro! the question" #xB+ ' $By%#xB+ ( $By% & 1$ #$%#xB+ ( $By% & 1$ xB+ ( $By & * We now have two equations with x and y" xB+ ' $By & $ #fro! this state!ent% xB+ ( $By & * #fro! the substitution above% 3o!bine the two equations #b adding%, then si!plif " #$%#xB+% & . #the y ter!s cancelled% xB+ & 4 x & 1$ The correct answer is / $+.

We can deter!ine the value of #a ( b%$ in one of three wa s" b figuring out the su! of a and b, b deter!ining what a and b are separatel , or b deter!ining the value of a$ ( $ab ( b$ #which is the quadratic for! of our product of factors%. #1% I>-?66I3I9>T" 5fter !ultipl ing both sides b b we can deter!ine that ab & 15, but we know nothing else. #$% I>-?66I3I9>T" If we 6;I0 #a ' b%$ we can learn that a$ ' $ab ( b$ & 4. 5lternativel , this state!ent also indicates that #a ' b% & $ or 4$. 2owever, neither of these !anipulations allow us to deter!ine the su! of a and b, the respective values of a and b individuall , or the value of a$ ( $ab ( b$. #1% 5>/ #$% -?66I3I9>T" -tate!ent 1 tells us ab & 15. If we substitute this into the quadratic equation fro! the second state!ent, we can deter!ine the value of a$ ( b$ in the following !anner" a$ ' $#15% ( b$ & 4 a$ ' +) ( b$ & 4 a$ ( b$ & +4 If we know the value of a$ ( b$, and the value of ab we can deter!ine the value of a$ ( $ab ( b$. a$ ( $ab ( b$ & a$ ( b$ ( $ab = +4 ( $#15% & *4 The correct answer is 3

/".
The equation x$y$ & 1. ' +xy is reall a quadratic, with the xy as the variable. x$y$ ( +xy ' 1. & ) #xy ( *%#xy ' +% & ) xy & + or 4* 2owever, we are told that x and y are positive so xy !ust equal +. Therefore, x & +By and x$ & ,By$. 5lternativel , this is a TI3 #variable in choice% and can be solved b plugging nu!bers. If we plug a value for y and find the corresponding value of x, we can check the answers to see which one !atches the value of x. 0ooking at the values + and 1. in the equation, a y value of + !akes sense. x$#+%$ & 1. ' +#x%#+% ,x$ & 1. ' ,x ,x$ ' ,x ( 1. & ) x$ ' x ( $ & ) #x ( $%#x ' 1% & ) x & 1, 4$ :ut since x cannot be negative, x & 1 If we plug y & + into each of the answer choices, #3% and #/% both give an x value of 1. #5% 1B+ #:% $ #3% 1 #/% 1 #9% 4 We !ust now plug another value of y to decide between #3% and #/%. ?lti!atel , onl #/% represents the correct value each ti!e. The correct answer is /

$5. This equation can be !anipulated into a quadratic equation b squaring both sides"

y$ & +y ( 4 y$ ' +y ' 4 & ) This quadratic equation can be factored to #y ( 1%#y ' 4% & ) There are two possible solutions for y" 41 and 4. The product of the two possible solutions for y is 44. The correct answer is 5 $*. We know that the su! of the cubes of a and b is ." a+ ( b+ & .. We also know that a* ' b* & 14. ?sing our knowledge of the quadratic te!plate for the difference of two squares, x$ ' y$ & #x ( y%#x ' y%, we can rewrite a* ' b* & 14 as follows" #a+%$ ' #b+%$ & 14 #a+ ' b+%#a+ ( b+% & 14 -ubstituting for a+ ( b+ gives" #a+ ' b+%#.% & 14 #a+ ' b+% & 14B. & 8B4 The correct answer is /

$8. ;ne wa to answer this question is to substitute 1Bx for x and 1By for y in the e7pression, then si!plif the resulting e7pression. 1Bx ( 1By 1Bx ' 1By Multipl the nu!erator and the deno!inator b xy to eli!inate the fractions. xy#1Bx ( 1By! #y ( x% & xy#1Bx ' 1By! #y ' x% -ince this is not one of the answer choices, it is necessar to si!plif further. With the knowledge that y ( x & x ( y and y ' x & 4#x ' y%, it can be stated that # y + x% #x ( y% &4 # y x% #x ' y% The correct answer is 5

$.. When a proble! involves variables raised to the fourth power, it is often useful to represent the! as a square of another square, since this approach will allow us to appl !anipulations of squares. 5lso note that since we are dealing with high e7ponents, the approach of plugging nu!bers would prove ti!e4consu!ing and prone to error in this case.

Therefore, let1s use algebra to solve this proble!. >ote that ,x4 ' 4y4 & #+x$%$ ' #$y$%$. We can represent this e7pression using the for!ula for the difference of two squares" ,x4 ' 4y4 & #+x$%$ ' #$y$%$ & #+x$ ( $y$%#+x$ ' $y$%. 0etGs use this shortcut to si!plif the equation" ,x4 ' 4y4 & +x$ ( $y$ #+x$ ( $y$%#+x$ ' $y$% & #+x$ ( $y$% 5t this step, our first instinct !a be to divide both sides of the equation b #+ x$ ( $y$%. =e!e!ber that in order to divide both siCes of the equation b an algebraic e7pression, we need to know that the value of this e7pression is not equal to Cero, since dividing b Cero results in an undefined outco!e. : looking again at the question, we see that 7 and are both non4Cero integers, so #+ x$ ( $y$% cannot be equal to Cero. Therefore, we can indeed si!plif the equation further b dividing out the #+ x$ ( $y$% fro! each side" #+x$ ' $y$% & 1 +x$ & $y$ ( 1 x$ & #$y$(1%B+ >ote that we could also have found the correct answer b plugging in nu!bers at this stage #since we have eli!inated the high e7ponents in the equation%. 6or e7a!ple, if we plug in & $ to the equation #+x$ ' $y$% & 1, we see that x$ & +. >ow we can plug & $ into each of the answer choices to find that onl 3 also gives us x$ & +. 6inall , if we are careful, we !ight also see that answer choices 5 and : cannot be correct because the are negative, and no non4Cero integer squared can equal a negative nu!ber. The correct answer is 3 $,. To find the ratio of r to s, we need to be able to solve 9IT29= for rBs ;= for r and s independentl . #1% I>-?66I3I9>T" The equation provided in state!ent 1 cannot be rewritten in the for! rBs & so!e value. #$% I>-?66I3I9>T" The equation provided in state!ent $ can be si!plified as follows" r$ ' s$ & 8 #r ( s%#r ' s% & 8. 2owever, this cannot be rewritten in the for! rBs & so!e value. #1% 5>/ #$% -?66I3I9>T" We can substitute the infor!ation fro! state!ent #1% in the equation fro! state!ent $ as follows" #r ( s%#r ' s% & 8. #8%#r ' s% & 8. r ' s & 1. 5dding this equation to the equation fro! the first state!ent allows us to solve for r. #r ' s & 1% ( #r ( s & 8% $r & . Thus, r = 4. If r is 4, then s !ust be +. The ratio of r to s is 4 to +. The correct answer is 3

+). z is the difference between the nu!ber of !en and the nu!ber of wo!en in the choirH hence z = x y. In order to answer the question Dwhat is zJE, we need to be able to deter!ine either the value of the quantit x y, or the values of both x and y fro! which quantit x y can be co!puted. #1% -?66I3I9>T" 5fter adding , to both sides of the equation, we get xU ' $xy ( yU & ,. -ince we are interested in the variables x and y, it would be helpful to rearrange the e7pression x" ' $xy ( yU into an e7pression that contains ter!s for x and y individuall . This suggests that factoring the e7pression into a product of two su!s is in order here. -ince the coefficients of both the xU and the y" ter!s are 1 and the coefficient of the xy ter! is negative, the !ost logical first guess for factors is #x ' y%#x ' y% or #x ' y%U. #We can quickl confir! that these are the correct factors b !ultipl ing out # x ' y%#x ' y% and

verif ing that this is equal to xU ' $xy ( yU.% 2ence, we now have #x y%" & , or x y & + or 4+. -ince the sti!ulus states that z or x y is a ph sical quantit #Dthere are z !ore !en than wo!enVE%, the onl answer that !akes logical sense is x y & +. #$% I>-?66I3I9>T" 5fter adding $$5 to both sides of the equation, we get xU ( $xy ( yU & $$5. -ince we are interested in the variables x and y, it would be helpful to rearrange the e7pression x" ( $xy ( yU into an e7pression that contains ter!s for x and y individuall . This suggests that factoring the e7pression into a product of two su!s is in order here. -ince the coefficients of both the xU and the y" ter!s are 1 and the coefficient of the xy ter! is positive, the !ost logical first guess for factors is # x ' y%#x ' y% or #x ' y%U. #We can quickl confir! that these are the correct factors b !ultipl ing out # x ' y% #x ' y% and verif ing that this is equal to xU 4 $xy ( yU.% 2ence, we now have #x + y%" & $$5 or x + y & 15 or 415. 9ven if we could pinpoint the value of x + y to one of those two values, this knowledge would not give us an insight as to the value of the quantit x 4 y, or the values of x and y individuall . The correct answer is 5 +1. In this proble!, we are given the infor!ation to set up the following three equations with three unknowns" x & zB4 x + y + z & $* y & $z ?sing the !ethod of substitution, we can now solve for each of the three unknowns. -ince the first equation, x & zB4, is an equation for x in ter!s of z, and the third equation, y & $z, is an equation for y in ter!s of z, we can replace x and y in the second equation b their equivalent e7pressions as follows" x + y + z & #z/4% ( #$z% ( z & $* We are left with an equation with just one unknown z. We can now solve for C" #z/4% ( #$z% ( z & $* zB4 ( .zB4 ( 4zB4 & 1+zB4 & $* zB4 & $ z&. >ow that we know z, we can easil solve for y, then co!pute y + z #note" we can also solve for x, but since we are not interested in the value of x there is no need to do so%" y & $z & $#.% & 1* y + z & 1* ( . & $4 -ince the largest factor of an nu!ber is the nu!ber itself, the largest factor of the su! of y and z is $4. The correct answer is 9

+$. -olve the original equation for b" $ ( 5a ' bB$ & +c $ ( 5a ' +c & bB$ 4 ( 1)a ' *c & b Wnowing the value of 1)a ' *c will allow us to calculate the value of b. -o, the rephrased question beco!es" LWhat is 1)a ' *cJL #1% I>-?66I3I9>T" Wnowing the su! of a and c is not enough to deter!ine the value of 1)a ' *c.

6or e7a!ple, if a & 1) and b & +, then 1)a ' *c & 1)#1)% ' *#+% & .$. 2owever, if a & * and b & 8, then 1)a ' *c & 1)#*% ' *#8% & 1.. #$% -?66I3I9>T" Manipulating the equation gives us the following" 41$c & 4$)a ( 4 $)a ' 1$c & 4 1)a ' *c & $ The correct answer is : ++. #1% I>-?66I3I9>T" We can start !anipulating this equation b !ultipl ing both sides b xy. 2owever, we see that the value of xy depends on the values of x and y.

The value of xy changes according to the values of x and y. #$% -?66I3I9>T" If we !anipulate this equation and solve for xy, we co!e up with a distinct value for xy.

The correct answer is : +4. To si!plif a radical in the deno!inator of a fraction, ou !ust !ultipl the deno!inator b so!ething that will cause the radical to disappear. Rou !ust also !ultipl the nu!erator b this sa!e value so as not to change the value of the fraction. #In effect, b !ultipl ing the nu!erator and the deno!inator b the sa!e value, ou are !ultipl ing the entire fraction b 1.% What will cause the radical in deno!inator to disappearJ Multipl the deno!inator co!ple!ent, , as follows" b its

The correct answer is :

35. #1B5%X!Y#1B4%X1.&#1B5X!%Y#1B$X+*%&1B#5X!Y$Y$X+5%&1B$#5X!Y$X+ 5% 1B$#5X!Y$X+5%& 1B$#1)%X+5, !eans that 5X!Y$X+5&1)X+5. ;bviousl , ! is +5

+*. 1 Y 4X11 can be e7pressed at the for!at $Y1)Xn,5X$1 Y 4X115X$1 Y $X$$&$Y1)X$1. -o, n&$1 +8. We can rewrite +11 as ++ P ++ P ++ P +$. -ince 5$ #or $5% is quite close to ++ #or $8%, we can replace each ++ with 5$ since the question asks us to appro7i!ate. The e7pression beco!es 5$. ( # 5$ P 5$ P 5$ P +$% & 5# or 5$. ( 5*#+$% & 5# We can factor out a 5* as follows" 5*#5$$ ( +$% & 5# -ince +$ #or ,% is insignificant co!pared to 5$$ #a huge nu!ber%, we can appro7i!ate the e7pression as" 5*#5$$%. # is appro7i!atel equal to $.. 5nother wa to look at this proble! is to realiCe that while +11 is a big nu!ber, it pales in co!parison to 5$.. The effect of adding +11 to 5$. will be !uch less than !ultipl ing 5$. b another 5 #i.e. 5$,%. To prove this, letGs look at two s!aller nu!bers, such as 5 4 #*$5% and ++ #$8%. When ou add $8 to *$5, the su! is !uch closer to 54 #*$5% then to 55 #+1$5%. The correct answer is 3. +.. The e7pression in the question can be si!plified" $x ( $x $# $x % $x(1 & & & $x4y(1 $y $y $ The question can be rephrased as Dwhat is x ' yJE since that would suffice to help us solve the e7pression. #1% -?66I3I9>T" The state!ent provides us with a value for x ' y. #$% I>-?66I3I9>T" The state!ent cannot be !anipulated to co!e up with a value for x ' y, nor can it alone provide a value for x and y. The correct answer is 5.

%#.
We can rephrase the question as LWhat is yJL or since we know that , LWhat is xJL

#1% -?66I3I9>T" 6ro! the question we know that , so x & y$. 5ccording to the state!ent, we also know that x & yx. If we set the two equations equal to one another, y$ & yx. x then !ust be equal to $. #There is another solution to this equation" :oth x and y are equal to 1. 2owever, the question ste! states that x is not equal to y so we can eli!inate this possibilit .% #$% -?66I3I9>T" If we take the cube root of both sides of the equation x+ & ., we find that x & $. The correct answer is /. 4). #1% I>-?66I3I9>T" This gives us a range of possible values for y. The low end of the range #1B5% is s!aller than 8B11, while the high end of the range #11B1$% is greater than 8B11. Thus, we cannot deter!ine whether y is greater than 8B11. #$% -?66I3I9>T" This gives us a range of possible values for y. The low end of the range #$B,% is s!aller than 8B11, and the high end of the range #.B1+% is als$ s!aller than 8B11. Thus, y cannot be greater than 8B11. The correct answer is :.

"1.
The ke to this question is to recogniCe the two co!!on algebraic identities" #x ( y%$ & x$ ( $xy ( y$ #x ( y%#x ' y% & x$ ' y$ In this question the x ter! is and the y ter! is , which !akes the two identities equal to"

If we si!plif the equation using these identities, we get"

The correct answer is 9 4$. #1% I>-?66I3I9>T" Rou cannot si!pl divide both sides of the equation b p to obtain p# & 1. The reason is that ou don1t know whether or not p is Cero 44 and re!e!ber, ou are not allowed to divide b CeroI Instead, ou !ust factor this equation. 6irst, subtract p fro! both sides to get" p#p ' p & ). Then, factor out a co!!on p to get" p#p# ' 1% & ). This !eans that either p# & 1 or p & ). #$% I>-?66I3I9>T" The sa!e process applies here as with state!ent #1%. =e!e!ber, ou should never divide both sides of an equation b a variable that could be Cero. 6irst, subtract # fro! both sides to get" #p# ' # & ). Then, factor out a co!!on # to get" ##p# ' 1% & ). This !eans that either p# & 1 or # & ). #1% 5>/ #$% I>-?66I3I9>T" Together we still donGt have enough infor!ation to solve. 9ither p# & 1 or both p and # are ). The correct answer is 9 4+. >otice that the identit in the nu!erator of the original fraction is written in the for! x$ y$, with x = 4x , and y = ,y$H to factor, rewrite it as #x + y%#x y%, or #4x$ + ,y$%#4x$ % ,y$%. We can also factor the right side of the first equation"
$

We can co!bine this equation with the other equation in the question" $x +y & + 4x ( +y & , *x & 1$ x&$ The correct answer is 3. 44. -ince f#x% & ax4 ' 4x$ ( ax ' +, f#b% & ax4 ' 4x$ ( ax ' + & ab4 ' 4b$ ( ab ' + 5>/ f#4b% & ax4 ' 4x$ ( ax ' + & a#4b%4 ' 4#4b%$ ( a#4b% ' + & ab4 ' 4b$ ' ab ' + Therefore" f#b% ' f#4b% & ab4 ' 4b$ ( ab ' + ' #ab4 ' 4b$ ' ab ' +% & $ab 5lternativel , we could have recogniCed that the onl ter! of the function that will be different for f#b% than for f#4b% is the Lax.L The other three ter!s are all unaffected b the sign of the variable. More succinctl , f#b% ' f#4b% !ust equal ab ' #4ab% & $ab. The correct answer is :

45. We are told that pZ# & p$ ( #$ ' $p#. In order for pZ# & p$ , the value of #$ ' $p# !ust equal ). We can solve this as follows" #$ ' $p# & ) ### ' $p% & ) The solution that would work for all value of p is if # & ). In plugging # & ) back into the original function, we get" p&# & p$ ( )$ 4 $p#)% & p$. The correct answer is 3

"5.
The e7pression in the question can be rewritten as 1 t$'+ .

#1% -?66I3I9>T" This state!ent can be rewritten as follows" 1 1 & +* t+'$ Therefore, t+'$ & +*. The onl positive integers that satisf this e7pression are t & 1 and ' = *. -ince we know the values of t and ', we can solve the e7pression in the question. #$% I>-?66I3I9>T" This state!ent can be rewritten as follows" t 1 t#'41% & ' & * There are !an possible values for t and '. 6or e7a!ple, t could be $ and ' could be 1$. 5lternativel , t could be 1 and ' could be *. -ince, there are !an possibilities for t and ', we are not able to solve the e7pression in the question. The correct asnwer is 5.

48. The ke to this proble! is recogniCing that the e7pression a ' b can be factored as the difference of two squares #x$ ' y$%, where The left side of the equation . can be factored as follows"

The correct answer is 3

"&.
#here is no useful rephrase of the question, so the best approach here is to analyze the statement to see what they tell us about the values of a, b, c, and d. &tatement - tells us that bLdL ) %'aLdL(. If we divide both sides by dL, we are left with bL ) %aL. Aemember that to find the factorial value of an integer, you multiply that integer by every positive integer smaller than it. &ince bL is % times greater than aL, it must be true that bL ) % x a x 'a , -( x 'a , +(... &ince bL is a factorial product and cannot have more than one % as a factor, it must be true that bL ) % x $ x + x -. #herefore, a ) $ and b ) %. Cut this tells us nothing about c or d. Insufficient. &tatement + tells us that ?.'bLcL( ) 'bLdL(. If we divide both sides by bL, we are left with ?.cL ) dL. &ince dL is ?. times greater than cL, dL could equal ?.L 'i.e., ?. x *2 x *>...(, and therefore d ) ?. and c = *2. "r dL could equal 'cL('$('%('*(, in which case cL must be +L and c ) + and d ) *. Insufficient. If we pool the information from both statements, however, we see that ?.' bLcL( ) %'aLdL(, which yields -*'bL cL( ) 'aLdL(. If we try this equation with a ) $, b ) %, c ) *2, and d ) ?., we get -*'%L*2L( ) '$L?.L( or ?.'$L *2L( ) '$L?.L(, which is the same as $L?.L ) $L?.L. &o these four values are possible. If we try the equation with a ) $, b ) %, c ) +, and d ) *, we get -*'%L+L( ) '$L*L( or '$('*('%L+L( ) '$L*L(, which is the same as *L$L ) $L*L. &o these four values are possible as well. &ince the value of c can be either + or *2 and the value of d can be either * or ?., we cannot answer the question definitively.

#he correct answer is H. %2. #he simplest approach to this problem is to pic numbers. Bet's say that x ) -. We can plug in - for x in :

:nd we can plug in - for x in

#herefore, for x, the only one to give

will be equal to as an answer is ::

. If we evaluate each choice by plugging in -

:lternatively, we can solve algebraically.

4irst, let's calculate the value of

. We can simplify this in terms of

!ow let's calculate the value of

. We can simplify this in terms of

We can now see that

is equal to the following:

#he correct answer is :.

50.
4irst, simplify the numerator by letting x = ab. #hen the numerator can be simplified as follows:

&ubstituting ab bac in for x, the original equation now loo s li e this:

In order for the fraction to have a value of ., the numerator must have a value of .. #hus, ab can be equal to ., ,?, or $. Dowever, since we are told that a and b are both nonzero integers, ab cannot be . and it must be equal to ,? or $. #herefore, a and b must be integer factors of ,? or $. #hus it would appear that: b can be equal to +, if a ) ,$ b can be equal to $, if a ) ,+ b can be equal to $, if a ) Dowever, a cannot be equal to ,+ or -, since this would ma e the denominator equal to . and leave the fraction undefined. #his leaves one option: a ) ,$ and b ) +. #he correct answer is : 'I only(: the variable b can be equal to +, not $ or %.

51. This proble! can be solved either algebraicall or b picking nu!bers. If the greater of the two integers is x, then the two integers can be e7pressed as x ' 1 and x. The su! of the reciprocals would therefore be

1 x'1

(1 x

x x#x ' 1%

(x'1 x#x ' 1%

$x ' 1 x$ ' x 5$. 6irst, look at state!ent #1% b itself. y & x #x 4 +% #x ( +% /istributing the right side of the equation" y & x #x$ ' ,% y & x+ ' ,x -ubtract ever thing on the right fro! both sides to get" y ' x+ ( ,x & ), which al!ost looks like the e7pression in the question. To !ake the left side of the equation !atch the question, subtract . x fro! both sides" ' x+ ( x & 4.x We would be able to answer the question if onl we knew the value of x, but that infor!ation is not given. -tate!ent 1 is not sufficient. -econd, look at state!ent #$% b itself. y & 45x -ince the question asks about a co!plicated e7pression of 7Gs and Gs, the si!plest wa to see if state!ent #$% is sufficient is to tr to !ake one side of the equation in state!ent #$% !atch the question, then tr to si!plif the other side of the equation to a single value. -ince the question asks about the value of y ( x+ ( x, and state!ent #$% has on the left side of the equation, add the D!issingE x+ ( x to both sides of the equation in state!ent #$%. y ( #x+ ( x% & 45x ( #x+ ( x% y ( x+ ( x & x+ 4 4x y ( x+ ( x & x #x$ ' 4% We would be able to answer the question if onl we knew the value of x, but that infor!ation is not given. -tate!ent $ is not sufficient. 6inall , look at both state!ents together. -ince both give e7pressions for y, set the right sides of each state!ent equal to each other" 45x & x #x ' +% #x ( +% 45x & x #x$ ' ,% 457 & x+ ' ,x ) & x+ ' 4x

) & x #x$ ' 4% ) & x #x ' $%#x ( $% -o, there are three solutions for x" [), $, or 4$\. 5t first, the state!ents together !ight see! insufficient, since this ields three values. 2owever, the question is not asking the value of x, rather the value of y ( x+ ( x. It is a good idea to find the value of y for each x value, then solve for the e7pression in the question. When x & ), y & ) and y ( x+ ( 7 & ) ( ) ( ) & ) When x & $, y & 41) and y ( x+ ( x & 41) ( . ( $ & ) When x & 4$, y & 1) and y ( x+ ( x & 1) 4 . 4 $ & ) The answer !ust be Cero, so the two state!ents together are sufficient. The correct answer is 3.

5+. The ke to solving this proble! is to recogniCe that the two given equations are related to each other. 9ach represents one of the ele!ents in the co!!on quadratic for!"

=ewrite the given equation as follows" Then, notice it1s relationship to the second given equation"

The second equation is in the for!

, while the first equation is in the for!

-ince we know that and that .. This gives us a third equation"

and that

we can solve for

, which !ust equal

5dding the second and third equations allows us to solve for x as follows"

Klugging this value for x into the first equation allows us to solve for y + z as follows"

The question asks for the value of x + y + z. If x = 1$ and y + z = $, then x + y + z = 1$ ( $ & 14. The correct answer is 9. 54. If we square both sides of the equation, we get z$ & *zs ' ,s$. We can now put the quadratic in standard for! z$ ' *Cs ( ,s$ & ) and factor #z ' +s%$ & ). -ince z ' +s & ), z & +s. This question can also be solved as a TI3 b plugging a value for the variable s. If we sa s & $, and plug this value into the equation after it was squared, we get" z$ & 1$z ' +* This can be written in standard for! and factored" #z ' *%$ & ), which !eans that z & *. >ow we can see which answer choice#s% ield * as the value for z when we plug in s & $. ;nl answer choice : works, +#$% & *. The correct answer is :. 55.

5*.

The left side of the equation ' bx%.

fits the for! a$x ' b$x, which can be factored as #ax ( bx%#ax

Thus, we can rewrite the equation as -ince 5nother wa to solve for , one wa to solve for is to find the value of a and b.

;= is to find the value of .

#1% -?66I3I9>T" We can subtract root + fro! both sides of the equation to obtain

& $.

#$% I>-?66I3I9>T" We can plug a & *4 into the original equation to solve for b. #*4%$B+ ' b$B+ & 1$ 1* ' b$B+ & 1$ b$B+ & 4 b & 4+B$ & (B4 . With a single value for a but two values for b, there are two solutions to the question. The correct answer is 5, -tate!ent #1% 50;>9 is sufficient to answer the question, but state!ent #$% alone is not. 57. To find the value of a4b we !ust either find the values of a and b and subtract the! or so!ehow !anipulate an equation so that we can solve directl for the co!bined e7pression a4b. -tate!ent #1% can be rewritten as"

-ince a, b, x and y are all positive integers, a4b !ust be an integer #although not necessaril a positive one%. It !a be easier to write this state!ent out in words" an integer # x! raised to so!e integer power #a4b% !ust equal three. -ince + is a pri!e nu!ber, it !ust follow that the base x is also +. If x were so!ething other than +, there would be no wa of raising it to an integer power and co!ing up with +. 9ven the base ,, which is co!posed of onl +1s, would need to be raised to a fractional e7ponent #i.e. square root & power of 1B$% to co!e up with +. If x !ust be +, we can set up the following equation" + a4b & +1. It follows that a4b & 1 and state!ent #1% is -?66I3I9>T. The answer !ust be 5 or /. -tate!ent #$% can be dealt with in a si!ilar !anner"

>otice, however, that the e7pression a4b now equals 4, which is not a pri!e nu!ber. :ecause 4 can be e7pressed as 41 or $$, the base y and the e7ponent a4b do not have fi7ed values. -tate!ent #$% is I>-?66I3I9>T and the correct answer is 5. 58.

When a bino!ial is e7panded, the nu!ber of ter!s is alwa s one !ore than the e7ponent of the bino!ial. 6or e7a!ple, . There are three ter!s and three is one !ore than two, the e7ponent. In this case, we have si7 ter!s, which !eans the value of 7 !ust be 5. We can now rewrite the e7pression in the question b substituting 5 for 7 as follows"

The question asks for the value of yz. To answer this we need to figure out the values of the coefficients y and z. 0etGs consider the coefficient in an easier e7pression such as

>otice that the coefficient $ represents the nu!ber of wa s that one a and one b can be !ultiplied together" either ab or ba. This is akin to counting the nu!ber of per!utations of $ unique ele!ents" $I & $. -i!ilarl , in , the coefficient + represents the nu!ber of wa s ou can !ultipl together two of the sa!e ter! and one of the other" aab, aba, baa or bba, bab, abb. This is akin to counting the nu!ber of per!utations of $ identical ele!ents and 1 unique one" In the case at hand, , we can figure out the value of the coefficient y, b counting the nu!ber of per!utations of 4 as and 1 b #or 4 bGs and 1 a%"

, so

& 5.

To figure out the value of the coefficient of z, we need to count the nu!ber of per!utations of + a1s and $ b1s #or $ bGs and + as%"

, so C & 1). Therefore, the value of yz is #5%#1)% & 5). The correct answer is 9. 5 . We are given an equation with two variables and asked to find the product of the variables. 5t first glance, it !a look i!possible to solve this equation for 7 and , since we have two variables and onl one equation. ;n top of that, the variables appear onl as e7ponents. 5nd, to pile it on, each answer choice has so !an factors that it would be totall i!practical to start b plugging in nu!bers. 2owever, using a co!bination of algebra and logic, we can figure out the values of 7 and and then find their product. 6irst, let1s rewrite the given equation so that all the variables are !oved to one side"

>ow, so!e logic is necessar to finish up.

We know that that both ter!s, -ince

and

, are positive. and !ust each be less than 5.

!ust equal 5, we also know that

We can now list all the possibilities, b testing s!all integer values for . We onl have to test a few values because we know that is 4, then !ust be less than 5, which !eans that can onl be 1, $ or +. #If would be ., which is greater than 5.%

If & 1, then , which !eans that two ter!s !ust be equal to 5%. 2owever, since 7 and !ake equal to 4.

!ust equal 4 #re!e!ber, the su! of the are positive integers, there is no wa to

If

& $, then

, which !eans that

!ust equal +. 2owever, since 7 and equal to +.

are

positive integers, there is no wa to !ake

If & +, then , which !eans that +, we can solve for 7 as follows"

!ust equal 1. This is possible. ?sing

&

The onl possible solution is 6!.

& + and 7 & 4. Therefore, 7 & #4%#+% & 1$. The correct answer is 9.

6ro! the proble! state!ent, we know that . We also know that the digits b c e and f are integers fro! ) to , and that the digits a and ( are integers fro! 1 to , #the cannot be ) since the are in the hundreds place%.

6or the state!ent above to be true, Therefore, .

!ust equal

, and

-ince the onl difference between abc and (ef is in the units digits, the difference between these three4digit nu!bers is equal to , or 1. The correct answer is 5. 61. In this t pe of proble!, the easiest thing to do is to e7press both sides of the equation in ter!s of pri!e nu!bers. The left side of the equation is alread e7pressed in ter!s of pri!e nu!bers, so we need to start b rewriting the right side of the equation in ter!s of pri!e nu!bers"

Thus, the given equation can be rewritten as follows"

]uite a bit is revealed b putting the equation in this for!. The right side of the equation, which we will call the DtargetE, is co!prised onl of $Gs and 5Gs. 0ooking at the left side of the equation, we see that we have x nu!ber of $Gs and y nu!ber of 5Gs along with so!e factor z. This unknown factor z !ust be co!prised of onl $Gs, onl 5Gs, so!e co!bination of $Gs and 5Gs, or it !ust be 1 #i.e. with no pri!e factors%. This is because an other pri!e co!ponents of C would ield a product that is different fro! the target. The question asks us to solve for xy, which we can certainl do b deter!ining the values of both x and y. -ince x and y si!pl tell us the nu!ber of $Gs and 5Gs, respectivel , that will be contributed toward the product on the left side of the equation #e.g., if x & $ and y & +, there are two $Gs and three 5Gs toward the product%, we can look at this question in a slightl different light. The onl other contributor to the final product on the left side is z. If we knew how !an $Gs andBor 5Gs that z contributed to the product, this would be enough to tell us what x and y are. 5fter the inclusion of z, an surplus or deficit of $Gs would have to be covered b x and an surplus or deficit of 5Gs would have to be covered b y. In other words, the question what is xy can be rephrased as what is zJ In state!ent #1% we are told that z & $), which is sufficient to answer our rephrased question. @ust to illustrate, this state!ent !eans that z provides the product with two additional $Gs and an additional 5, since for x and y as follows" on the left side of our equation . We can use this infor!ation to solve

In state!ent #$% we are given the nu!ber of $Gs contributed b the e7pression $ x on the left side of the equation. To hit the target, z !ust contain e7actl two $Gs, since . :ut what about 5GsJ Is the e7pression 5y the onl source of 5Gs or is z co!posed of 5Gs as wellJ We have no wa of knowing so we cannot find the value of z #or y%.

The correct answer is #5%" -tate!ent #1% alone is sufficient, but state!ent #$% alone is not sufficient. 62. ;ne of the !ost effective wa s to begin solving proble!s involving e7ponential equations is to break down bases of the e7ponents into pri!e factors and co!bine e7ponents with the sa!e base. 6ollowing this approach, be sure to si!plif each state!ent as !uch as possible before arriving at the conclusion, since difficult proble!s with e7ponents often result in unobvious outco!es. #1% I>-?66I3I9>T" While this state!ent gives us the value of x, we know nothing about y and cannot deter!ine the value of xy. #$% -?66I3I9>T" #1$.x%#*x + y% & #4.$x%#+%x% #$8%x#$ P +%x + y & #$4 P +%$x#+%x% #$8x%#$x ( y%#+x ( y% & #$.x%#+%$x#+%x% #$.x + y%#+x + y% & #$.x%#+%$x % x #$.x%# $y%#+x%#+y% & #$.x%#+%x # $y%#+y% & 1 #$ P +%y & 1 *y & 1 y&) -ince y & ) and x is not equal to Cero #as stated in the proble! ste!%, this infor!ation is sufficient to conclude that xy & x) & 1. The correct answer is :.

5%. )n order to evaluate the function f+n-, simply substitute the value of n for every instance of n on the rightGhandGside of the definition of the function. :or e*ample, for n , ", f+"- , f+%- C ". <ote that the value of f+n- is dependent on the value of f+n C 1-. Therefore, in order to find f+"-, we must ;now f+%-. 'o one way of rephrasing the question is WWhat is the value of f+%-QX Mowever, let6s suppose we don6t ;now f+%- but we ;now f+/-. 'ince, f+%- , f+/- C %, we can calculate the value of f+%- from f+/-, then f+"- from f+%-. Aontinuing this logic, if we ;now the value of f+1-, we can calculate the value of f+/- from f+1-, then f+%from f+/-, and then f+"- from f+%-. )t is apparent that if we ;now the value for f+iwhere i is any integer less than ", we can eventually get to the value of f+"- by successive calculating f+n- for increasing n6s. We can also rearrange the equation f+n- , f+n C 1- C n to f+n C 1- , f+n- 4 n. 'o if we ;now f+5-, then f+5 C 1- or f+"- , f+5- 4 5. Mence, given f+5-, we can calculate f+"-. Bsing similar logic as above, if we ;now f+5-, we can calculate the value of f+5- from f+5-, then f+"- from f+5-. We can see that we ;now the value of f+i- for any integer i greater than ", we can eventually get to the value of f+"- by successively calculating f+n C 1- for decreasing n6s. Therefore, if we ;now the value of f+i- for any one specific value of i, we can get to the value of f+"-S hence, the question can be restated as W hat is the !al"e #$ $%i& $#' any s(e)i$i) inte*e' i+, +1- 'B::)A)?<T. 'ince we are given the value of f+i- for the specific integer i , %, it follows that f+"- can be calculated.

+/- 'B::)A)?<T. 'ince we are given the value of f+i- for the specific integer i , 5, it follows that f+"- can be calculated. The correct answer is $.

5". 'ince we ;now the value of YG!Y , %, we can plug p , G! into our formula +G!-%a + +G!-3 C 1 , % G%"%a C !3 , % G%"%a C !3 , " We are as;ed to solve for Y!Y. )f we plug ! into our formula, we get +!-%a 4 +!-3 < 1 , Q %"%a 4 +!-3 C 1 , Q To figure this out, we would need to ;now the value of %"%a 4 !b. :rom the first equation we ;now that G%"%a < !3 , ". .y multiplying both sides by negative one, we see that %"%a 4 !3 , G". %"%a + !3 C 1 , Q G" C 1 , G5 The correct answer is ?. 55. )t helps to recogniJe that this is a quadratic equation problem presented as a function problem. What we are essentially being told is that 5 and G% are the two Jeros for the equation x/ 4 3x 4 (. )n other words, 5 and G% are the two solutions to the equation x/ 4 3x 4 ( , 0. .ecause solutions are always the opposites of the factor numbers, we ;now that our equation in factored form is +x C 5-+x 4 %- , 0 which, when :=)2ed, becomes x/ C %x C 1& , 0 'o 3 , G%, and ( , G1&. Therefore, 3 4 ( , G/1. (lternatively, we can find the values of 3 and ( by using substitution. )f f+5- , 0, then 5/ 4 3+5- 4 ( , 0 53 4 ( , G%5

)f f+G%- , 0, then +G%-/ 4 3+G%- 4 ( , 0 G%3 4 ( , G# We can combine the two equations and solve. )n this method, we add or subtract the two equations to eliminate one of the variables. )n this problem, we can use subtraction to eliminate (. 53 4 ( , G%5 C +G%3 4 ( , G##3 , G/! 3 , G% We can substitute G% for 3 into either of the above equations to get ( , G1&. )t follows that 3 4 ( , G/1. The correct answer is $.

55. This problem can be solved algebraically or by plugging in the answer choicesS both methods are shown below. %lge3ra .ecause there are square root signs on both sides of the equation, we can square both sides to get rid of them, which leaves us with " 4 x1D/ , x 4 /. x1D/ is the same thing as , so our ne*t step is to isolate the radical sign and then square both sides again. =nce we do this, we can solve for x. ,xC/ x , +x C /-/ x , x/ C "x 4 " 0 , x/ C 5x 4 " 0 , +x C "-+x C 1x can equal either " or 1S try each to determine which one is the solution to the original equation x," , , , so x can equal ".

x,1 =nly x , " wor;s.

, so x cannot equal 1.

Plugging in the ans!ers 'ince the numerical answers represent a possible value for x, we can also plug them into the equation and see which one wor;s. @emember that we can stop when we find the right answer and itEs also best to start with answer choice A. )f A does not wor;, we can sometimes

determine whether we want to try a larger or smaller number ne*t, thereby saving some time.

+?- 'ince $ wor;s, the answer can be determined. :(2'? The correct answer is $. 5!. The equation in question can be rephrased as follows x/y C 5xy 4 #y , 0 y+x/ C 5x 4 #- , 0 y+x C %-/ , 0 Therefore, one or both of the following must be true y , 0 or x,% )t follows that the product xy must equal either 0 or %y. This question can therefore be rephrased WWhat is yQX +1- )<'B::)A)?<T This equation cannot be manipulated or combined with the original equation to solve directly for x or y. )nstead, plug the two possible scenarios from the original equation into the equation from this statement )f x , %, then y , % 4 x , % 4 % , 5, so xy , +%-+5- , 1&. )f y , 0, then x , y C % , 0 C % , G%, so xy , +G%-+0- , 0. 'ince there are two possible answers, this statement is not sufficient. +/- 'B::)A)?<T )f x% O 0, then x O 0. Therefore, x cannot equal %, and it follows that y , 0. Therefore, xy , 0. The correct answer is ..

También podría gustarte